聪明文档网

聪明文档网

最新最全的文档下载
当前位置: 首页> 英语专四历年真题及答案(1996-2011)

英语专四历年真题及答案(1996-2011)

时间:2014-05-27 10:51:56    下载该word文档

TEST FOR ENGLISH MAJORS (2013)

1. According to the conversation, an example of “Christmas trimmings” could be

A. presents. B. fruits C. sauce D. meat

2. A Christmas lunch would include all the following EXCECT

A. roast turkey B. sweet potatoes C. meat D. carrots

3. Why did Helen come to Rob’s house?

A. She wanted to talk to Bob. B. She had come to help Bob.

C. She had been invited to lunch. D. She was interested in cooking.

Questions 4 to 7 are based on the following conversation. At the end of the conversation you will be given 20 seconds to answer the questions.

4. Why did the woman phone the club?

A. She wanted to know more about it. B. She was a new comer and felt lonely.

C. She wanted to learn a new language. D. She was interested in social activities.

5. We learn from the conversation that the club

A. mainly organize language activities. B. accepts members from local students.

C. has been set up for a long time. D. is increasing its membership.

6. According to the conversation, the woman might come to practice German on

A. Wednesday. B. Tuesday. C. Monday. D. Friday.

7. What is the man going to do after the conversation?

A. Call up the woman for her address. B. Wait for the woman to call him again.

C. Mail the woman some information. D. Wait for the woman to pick up a form.

Questions 8 to 10 are based on the following conversation. At the end of the conversation you will be given 15 seconds to answer the questions.

8. According to the woman, what actually makes her job difficult?

A. Difficult questions from interviewees.

B. Embarrassing requests from interviewees.

C. Lack of professional background.

D. Lack of interviewing skills.

9. The woman uses all the following adjectives when talking about attending job fairs EXCEPT

A. prospective. B. useful. C. important. D. tiring.

10. We learn from the conversation that the woman

A. works better at job fairs. B. prefers honest people.

C. often works on her own. D. is experienced in her work.

SECTION B PASSAGES

In this section, you will hear several passages. Listen to the passages carefully and then answer the questions that follow.

Questions 11 to 13 are based on the following passage. At the end of the passage you will be given 15 seconds to answer the questions.

11. According to today’s weather forecast, which part of Europe has dry weather?

A. Scandinavian mountain. B. Northwestern Europe.

C. Northern Europe. D. Southern Europe.

12. In which part of Europe does the weather stay both fine and cool?

A. Southern Europe. B. Northern Europe. C. Eastern Europe. D. Northwestern Europe.

13. In which region will the weather change tomorrow?

A. Northern parts of the Mediterranean. B. Eastern parts of the Mediterranean.

C. Central parts of the Mediterranean. D. Southern parts of the Mediterranean.

Questions 14 to 17 are based on the following passage. At the end of the passage you will be given 20 14. According to the passage, what benefit can technology bring to people?

A. Closer contact with modern devices.

B. Greater changes in social organization.

C. Better understanding of mass media.

D. More useful information to better their life.

15. The speaker questions about everybody’s access to technological advances. The main reason is

A. illiteracy. B. poverty. C. food shortage. D. ignorance.

16. According to the UN plan, all the following will be achieved within ten years EXCEPT

A. giving everyone a radio or TV.

B. starting to carry out the scheme in ten years.

C. offering internet service to more people.

D. providing more job opportunities.

17. What could be topic of the passage?

A. Growth in telecommunications. B. Technology and the developing world.

C. Education and medical care. D. Building an information society.

Questions 18 to 20 are based on the following passage. At the end of the passage you will be given 15 seconds to answer the questions.

18. People in Latin America wear something ______ to express their hopes for wealth in the New Year.

A. new B. red C. white D. yellow

19. Which of the following New Year’s traditions signals friendship?

A. Throwing old dishes. B. Wearing something red.

C. Wearing something white. D. Eating round fruits.

20. Which of the following is NOT mentioned as one’s own New Year’s tradition?

A. Watching TV at home. B. Going to bed early.

C. Visiting friends. D. Running and shouting outside.

SECTION C NEWS BROADCAST

In this section, you will hear several news items. Listen to the passages carefully and then answer the questions that follow.

Questions 21 to 22 are based on the following news. At the end of each news item, you will be given 10 seconds to answer the questions.

21. What is happening to the schools in Fairfax County this school year?

A. 15 schools have started social studies.

B. 15 schools have used digital textbooks.

C. Students are ready to use electronic resources.

D. Digital textbooks are used for social studies.

22. With digital textbooks, schools have saved about ______ million dollars.

A. 1 B. 2 C. 3 D. 4

Questions 23 to 24 are based on the following news. At the end of each news item, you will be given 10 seconds to answer the questions.

23. Who found the suspicious item at the airport?

A. TSA agents. B. FBI agents. C. The police. D. Passengers.

24. Which of the following statement is INCORRECT?

A. The terminal was closed temporarily afterwards.

B. There was a thorough search inside the airport.

C. Passengers at the airport were safe and sound.

D. The security authorities identified the explosives.

Questions 25 to 26 are based on the following news. At the end of each news item, you will be given 10 seconds to answer the questions.

25. According to the news item, doctors use art therapy to treat the following problems EXCEPT

A. alcohol abuse. B. smoking. C. depression. D. schizophrenia.

26. Why did doctors introduce art therapy in the first place?

A. To prevent patients from smoking. B. To better understand patients.

C. To get patients occupied. D. To teach patients some skills.

Questions 27 to 28 are based on the following news. At the end of each news item, you will be given 10 seconds to answer the questions.

27. What is the main purpose of the new rules?

A. To reduce the number of pilots on duty.

B. To prevent pilots from working overtime.

C. To ensure an adequate amount of sleep.

D. To fix the amount of work for each pilot.

28. The Independent Pilots Association was unhappy about the new rules because they

A. had only covered cargo plane pilots.

B. had failed to cover all the pilots.

C. would be put into effect in two years.

D. would be too costly if implemented.

Questions 29 to 30 are based on the following news. At the end of each news item, you will be given 10 seconds to answer the questions.

29. Why is increase in livestock production necessary?

A. Because livestock production is highly efficient.

B. Because more people will become wealthier.

C. Because it may help double food production.

D. Because it has fewer ecological risks.

30. What does the word “challenge” mean in the news item?

A. Balance between human survival and ecology.

B. Conflict between less land and more production.

C. Difference between present and future needs.

D. Calls by environmental critics to consume less meat.

PART 3 CLOZE 15 MIN

  Everyone knows that taxation is necessary in a modern state: without  it, it (31) _____ not be possible to pay the soldiers and policemen who  protect us; (32) _____ the workers in government offices who (33) _____ our health, our food, our water, and all the other things that we cannot do for ourselves. (34) _____ taxation, we pay for things that we  need just (35) _____ we need somewhere to live and something to eat.   But (36) _____ everyone knows that taxation is necessary, different   people have different ideas about (37)____ taxation should be arranged.

 In most countries, a direct tax on (38) _____, which is called  income tax, (39) _____. It is arranged in such (40)______ that the  poorest people pay nothing, and the percentage of tax grows (41) ____ as the taxpayer's income grows. In some countries, for example, the tax on  the richest people (42)______ as high as ninety-five per cent!  (43) _____ countries with taxation nearly (44) _____ have indirect  taxation too. Many things imported into the country have to pay taxes or duties.” Of course, it is the men and women who buy these imported  things in the shops (45)______ really have to pay the duties, in the (46) ______ of higher prices. In some countries, (47) _____, there is a tax on  things sold in the shops. If the most necessary things are taxed, a lot   of money is (48) ____ but the poor people suffer most. If unnecessary   things (49)___ jewels and fur coats are taxed, less money is obtained  but the tax is (50)  ______    as the rich pay it.  

Probably this last kind of indirect tax, together with a direct tax on incomes which is low for the poor and high for the rich, is the best arrangement.  

PART 4 GRAMMAR & VOCABULARY     15 MIN

There are thirty sentences in this section. Beneath each sentence there are four words or  phrases marked A, B, C and D. Choose one word or phrase that best completes the sentence. Mark your answers on Answer Sheet Two.  

51. Facing the board of directors, he didn‟t deny ________ breaking the agreement.  

A. him

B. it

C. his 

D. its  

 52. Xinchun returned from aboard a different man. The italicized part functions as a (n)  ______.  

A. appositive (同位语)

B. object 

C. adverbial 

D. complement.  

53. Which of the following is a compound word (复合词)?  ______.  

A. Nonsmoker.  

B. Deadline.   

C. Meanness.   

D. Misfit.  

54. Which of the following sentences contains subjunctive mood?  ______.  

A. Lucy insisted that her son get home before 5 o’clock?  

B. She used to drive to work, but now she takes the city metro.   

C. Walk straight ahead, and don't turn till the second traffic lights.   

D. Paul will cancel his flight if he cannot get his visa by Friday.  

  

55. The following determiners(限定词) can be used with both plural and uncountable nouns  

 EXCEPT   ______.   

 A. more

B. enough.

C. many. 

D. such.  

56. Which of the italicized parts indicates CONTRAST?  ______.  

A. She opened the door and quietly went in.  

B. Victoria likes music and Sam is fond of sports.  

C. Think it over again and you‟ll get an answer.   

D. He is somewhat arrogant, and I don't like this.

57. Which of the following CANNOT be used as a nominal substitute(名词替代词)?  ______.  

A. Much.  

B. Neither.  

C. One.  

D. Quarter.  

 58. All the following sentences definitely indicate future time EXCEPT  ______.  

A. Mother is to have tea with Aunt Betty at four.  

B. The President is coming to the UN next week. 

C. The school pupils will be home by now.  

D. He is going to email me the necessary information.  

59. Which of the following sentences is grammatically INCORRECT?  ______.  

A. Politics are the art or science of government.  

B. Ten miles seems like a long walk to me.  

C. Mumps is a kind of infectious disease. 

D. All the furniture has arrived undamaged.  

60. Which of in the following phrases indicates a subject-predicate relationship?  ______.  

A. The arrival of the tourists  

B. The law of Newton  

C. The occupation of the island  

D. The plays of Oscar Wilde  

61. Which of the following italicized parts serves as an appositive?  ______.  

A. He is not the man to draw back.  

B. Tony hit back the urge to tell a lie.  

C. Larry has a large family to support.  

D. There is really nothing to fear.  

62. Which of the following is NOT an imperative sentence?  ______.  

A. Let me drive you home, shall I? 

B. You will mind your own business!  

C. Come and have dinner with us.  

D. I wish you could stay behind.  ‘

63. If it ______ tomorrow, the match would be put off.  

A. were to rain  

B. was to rain   

C. was raining   

D. had rained  

64. Which of the following sentences expresses a fact?  ______.  

A. Mary and her son must be home by now.  

B.Careless reading must give poor results.   

C. Its getting late, and I must leave now.   

D. He must be working late at the office.  

65. The following are all dynamic verbs(动态动词) EXCEPT  ______.   

A. remain. 

B. turn. 

C. write.    

D. knock.  

66. ____ to school life was less difficult than the pupil had expected.  

A. Adhering 

B. Adopting 

C.Adjusting 

D. Acquainting  

   

67. He is fed up with the same old dreary routine, and wants to quit his job. The underlined  part 

means _________.   

A. dull 

B. boring 

C. long 

D. hard  

   

68. At last nights party Larry said something that I though was beyond me. The underlined  

part means ________.   

A. I was unable to do  

C. I was unable to stop  

B. I couldn’t understand  

D. I couldn’t tolerate  

   

69. The couple ______their old house and sold it for a vast profit.  

 

A. did for 

B. did in 

C.did with 

D. did up  

   

70. Sally contributed a lot to the project, but she never once accepted all the ____ for  herself.  

 

A. credit 

B. attention  

C. focus 

D. award 

71. The child nodded, apparently content with his mothers promise. The underlined part  

means _________.  

A. as far as one has learnt  

B. as far as one is concerned  

C. as far as one can see  

D. as far as one is told  

  

72. The ________ that sport builds character is well accepted by people nowadays.  

A. issue 

B. argument 

C. point 

D. sentence  

  

73. Everyone in the office knows that Melinda takes infinite care over her work. The  

underlined part means ________.  

A. limited 

B. unnecessary 

C. overdue 

D. much  

  

74. The new measure will reduce the chance of serious injury in the event of an accident.  

The underlined part means _________.  

A. if an accident happens  

B. if an accident can be prevented 

C. before an accident  

D. during an accident  

  

75. Traditionally, local midwives would ________ all the babies in the area.  

A. handle 

B. produce 

C. deliver 

D. help  

  

76. No food or drink is allowed on the premises. The underlined part means ________.  

A. proposition 

B. advertisement 

C. building 

D. street 

  

77. The court would not accept his appeal unless ________ evidence is provided.  

A. conclusive 

B. definite 

C. eventual 

D. concluding  

  

78. As soon as he opened the door, a ________ of cold air swept through the house.  

A. flow 

B. movement 

C. rush 

D. blast  

  

79. She really wanted to say something at the meeting, but eventually ________ from it.  

A. prevented 

B. refrained 

C. limited 

D. restricted  

  

80. The couple told the decorator that they wanted their bedroom gaily painted. The  underlined part means _________.  

A. brightly 

B. light-heartedly 

C. cheerfully 

D. lightly 

PART V READING COMPREHENSION    25 MIN

  

In this section there are four passages followed by questions or unfinished statements,  each with four suggested answers marked A, B, C and D. Choose the one that you think is the  best answer. Mark your answers on Answer Sheet Two.  

  

TEXT A  

 

The art of public speaking began in ancient Greece over 2,000 years ago. Now, twitter,  instant messaging, e-mail, blogs and chat forums offer rival approaches to communication - but none can replace the role of a great speech.  

 The spoken word can handle various vital functions: persuading or inspiring, informing,  paying tribute, entertaining, or simply introducing someone or something or accepting  something.  Over the past year, the human voice has helped guide us over the ups and downs of what  was certainly a stormy time.  

Persuasion is used in dealing with or reconciling different points of view. When the  leaders met in Copenhagen in December 2009, persuasive words from activists encouraged them to commit themselves to firmer action.  Inspirational speeches confront the emotions. They focus on topics and matters that are  close to people's hearts. During wars, generals used inspiring speeches to prepare 

the troops  for battle.  

A speech that conveys knowledge and enhances understanding can inform us. The  information must be clear, accurate, and expressed in a meaningful and interesting way.(流行病)announced, the idea of swine flu many people. Informative speeches from World Health Organization officials helped people  to keep their panic under control so they could take sensible precautions.  

 Sad events are never easy to deal with but a speech that pays tribute to the loss of a loved  one and gives praise for their contribution can be comforting. Madonna's speech about   Michael Jackson,after his death, highlighted the fact that he will continue to live on through   his music.  

It's not only in world forums where public speaking plays an important role. It can also  be surprisingly helpful in the course of our own lives.  

If youre taking part in a debate you need to persuade the listeners of the soundness of  

your argument. In sports, athletes know the importance of a pep talk(鼓舞士气的讲话) before a match to inspire teammates. You yourself may be asked to do a presentation at  college or work to inform the others about an area of vital importance.  

 

On a more personal level, a friend may be upset and need comforting. Or you might be  

asked to introduce a speaker at a family event or to speak at a wedding, where your language  will be needed to move people or make them laugh.  

Great speaking ability is not something we're born with. Even Barack Obama works hard  

to perfect every speech. For a brilliant speech, there are rules that you can put to good use. To  learn those rules you have to practice and learn from some outstanding speeches in the past.  

   

81. The author thinks the spoken word is still irreplaceable because ______.  

A. it has always been used to inspire or persuade people.  

B. it has a big role to play in the entertainment business.  

C. it plays important roles in human communication.  

D. it is of great use in everyday-life context.  

  

82. Which of the following statements is INCORRECT about the role of public speaking?  _____

A. Speeches at world forums can lead to effective solutions to world problems.  

B. Speeches from medical authorities can calm people down in times of pandemics.  

C. The morale of soldiers before a battle can be boosted by senior officers' speeches.  

D. Speeches paying tribute to the dead can comfort the mourners.  

  

83. Public speaking can play all the following roles EXCEPT   ______.  

A. to convince people in a debate.  

B. to inform people at a presentation.  

C. to advise people at work.  

D. to entertain people at a wedding.  

  

84. According to the passage, which of the following best explains the author's view on  ______ great speaking ability?  

A. It comes from observing rules.  

B. It can be perfected with easy effort.  

C. It can be acquired from birth.  

D. It comes from learning and practice.

85. What is the main idea of the passage?  ______.   

A. Public speaking in international forums.  

B. The many uses of public speaking.   

C. Public speaking in daily life context.   

D. The rules of public speaking.  

  TEXT B  

Every business needs two things, says Skullcandy CEO Rick Alden: inspiration and   desperation. In 2001, Alden had both. He'd sold two snowboarding businesses, and he was  desperately bored. But he had an idea: He wanted to make a new kind of headphone.   I kept seeing people missing their cell phone calls because they were listening to  music, he explains. Then I'm in a chairlift, I've got my headphones on, and I realize  my phone is ringing. As 1 take my gloves off and reach for my phone, I think, It can't be  that tough to make headphones with two plugs, one for music and one for your cell phone.  Alden described what he wanted to a designer, perfected a prototype, and outsourced(外包)manufacturing overseas.  

Alden then started designing headphones into helmets, backpacks - anywhere that would  make it easy to listen to music while snowboarding. Selling into board and skate shops   wasn't a big research effort, he explains. Those were the only guys I knew!  

Alden didnt want to be a manufacturer. And by outsourcing, he'd hoped he could get the  

business off the ground without debt. But he was wrong. So he asked his wife, Can I put a  

(抵押贷款) on the house? She said, What is the worst thing that can happen? We  

lose the house, we sell our cars, and we start all over again. I definitely married the right  

woman!  

 

For the next two years, Alden juggled mortgage payments and payments to his  manufacturers. Factories won't ship your product till they get paid, he says. But it takes  four or five months to get a mortgage company so upset that they knock on your door. So we   paid the factory first.  

Gradually, non-snowboarders began to notice the colorful headphones. In 2006, the  company started selling them in 1,400 FYE (For Your Entertainment) stores. We knew that  nine out often people walking into that store would be learning about Skullcandy for the first   time. Why would they look at brands they knew and take home a new brand instead? We had  agreed to buy back anything we didn’t sell, but we were dealing with huge numbers. It’d kill  us to take back all the products

  

 Aldens fears faded as Skullcandy became the No. 1 headphone seller in those stores and   

tripled its revenue to $120 million in one year. His key insight was that headphones weren’t  

 gadgets; they were a fashion accessory. In the beginning, he says, that little white wire   that said you had an iPod ---that was cool. But now wearing the white bud means you’re just   like everyone else. Headphones occupy this critical piece of cranial real estate and are highly  visible.  

 

Today, Skullcandy is America's second-largest headphone supplier, after Sony. With 79 employees, the company is bigger than Alden ever imagined.  

86. Alden came up with the idea of a new kind of headphone because he  ______.  

 A. was no longer in snowboarding business.  

B. had no other business opportunities. 

C. was very fond of modern music.   

D. saw an inconvenience among mobile users.  

87. The new headphone was originally designed for ______.  

A. snowboarders.  

B. motorcyclists.   

C. mountain hikers.   

D. marathon runners.  

88. Did Alden solve the money problem?  ______.  

A. He sold his house and his cars.  

B. Factories could ship products before being paid.   

C. He borrowed money from a mortgage company.   

D. He borrowed money from his wife's family.  

   

89. What did Alden do to promote sales in FYE stores?  ______.  

A. He spent more money on product advertising.  

B. He promised to buy back products not sold.   

C. He agreed to sell products at a discount.

D. He improved the colour design of the product.  

90. Alden sees headphones as  ______.

A. a sign of self-confidence.  

B. a symbol of status.   

C. part of fashion.   

D. a kind of device.  

   

TEXT C  

I was standing in my kitchen wondering what to have for lunch when my friend Taj   called.  Sit down, she said.   I thought she was going to tell me she had just gotten the haircut from hell. I 

laughed   and said, It can't be that bad.  

 

But it was. Before the phone call, I had 30 years of retirement saving in a safe fund  with a 

brilliant financial guru(金融大亨).When I put down the phone, my savings were  

gone. I felt as if I had died and, for some unknown reason, was still breathing.   

Since Bernie Madoffs arrest on charges of running a $65 million Ponzi scheme, I've  

read many articles about how we investors should have known what was going on. I wish I  

 could say I had reservations about Madoff before the Call, but I did not.  

 On New Year's Eve, three weeks after we lost our savings, six of us Madoff people  gathered at 

Taj's house for dinner. As we were sitting around the table, someone asked, If  you could have 

your money back right now, but it would mean giving up what you have   learned by losing it, 

would you take the money or would you take what losing the money   has given you?  

 My husband was still in financial shock. He said, I just want the money back. I wasn't  certain where I stood. I knew that losing our money had cracked me wide open. I’d been  walking around like what the Buddhists call a hungry ghost: always focused on the bite that   was yet to come, not the one in my mouth. No matter how much I ate or had or experienced,   it didn’t satisfy me, because I wasn’t really taking it in, wasn't absorbing it. Now I was   forced to pay attention. Still, I couldn't honestly say that if someone had offered me the   money back, I would turn it down.  

 But the other four all said that what they were seeing about themselves was incalculable,  and 

they didn’t think it would have become apparent without the ground of financial  stability being 

ripped out from underneath them.  

My friend Michael said,I’d started to get complacent. It’s as if the muscles of my heart  started to atrophy(萎缩). Now theyre awake, alive—and I dont want to go back.  

Theseweren’t just empty words. Michael and his wife needed to take in boarders to  meet their expenses. Taj was so broke that she was moving into someone’s garage  apartment in three weeks. Three friends had declared bankruptcy and weren't sure where or   how they were going to live.  

   

91. What did the author learn from Tajs call? 

 A. had got an awful haircut.  

B. They had lost their retirement savings.   

C. Taj had just retired from work.   

D. They were going to meet for lunch.  

   

92. How did the author feel in the following weeks?  

A. Angry.  

B. Disappointed.   

C. Indifferent.  

D. Desperate.  

93. According to the passage, to which was she forced to pay attention

A. Her friends.  

B. Her husband.   

C. Her lost savings.   

D. Her experience.  

 

94. Which of the following statements is CORRECT about her friends?

A. Her friends valued their experience more.  

B. Her friends felt the same as she did.   

C. Her friends were in a better financial situation.   

D Her friends were more optimistic than she.  

 

95. What is the message of the passage? 

 A. Desire for money is human nature.  

 B. One has to be decisive during crises.  

C. Understanding gained is more important than money lost.  

D. It is natural to see varied responses to financial crises.  

   

TEXT D  

In the 19th century, there used to be a model of how to be a good person. There are all these torrents of passion flowing through you. Your job, as captain of your soul, is to erect  dams to keep these passions in check. Your job is to just say no to laziness, lust, greed, drug   use and the other sins.  

 These days that model is out of fashion. You usually can’t change your behaviour by  simply resolving to do something. Knowing what to do is not the same as being able to do it.  

 Your willpower is not like a dam that can block the torrent of self-indulgence. It's more like a   muscle, which tires easily. Moreover, you're a social being. If everybody around you is   overeating, youll probably do so, too.  

 The 19th-century character model was based on an understanding of free will. Today, we  

 know that free will is bounded. People can change their lives, but ordering change is not  

 simple because many things, even within ourselves, are beyond our direct control.  

 Much of our behaviour, for example, is guided by unconscious habits. Researchers at  

 Duke University calculated that more than 40 percent of the actions we take are governed by  

 habit, not actual decisions. Researchers have also come to understand the structure of   habits—cue, routine, reward.  

 You can change your own personal habits. If you leave running shorts on the floor at  

 night, that'll be a cue to go running in the morning. Dont try to ignore your afternoon snack  

 craving. Every time you feel the cue for a snack, insert another routine. Take a walk.  

 Their research thus implies a different character model, which is supposed to manipulate  

the neural(神经系统的)networks inside.  

 To be an effective person, under this model, you are supposed to coolly examine your  

 own unconscious habits, and the habits of those under your care. You are supposed to devise  

 strategies to alter the cues and routines. Every relationship becomes slightly manipulative,   including your relationship with yourself. You're trying to arouse certain responses by   implanting certain cues.  

 This is a bit disturbing, because the important habitual neural networks are not formed  

 by mere routine, nor can they be reversed by clever cues. They are burned in by emotion and  

strengthened by strong yearnings, like the yearnings for admiration and righteousness. If you think you can change your life in a clever way, the way an advertiser can get you  to buy an air freshener, you're probably wrong. As the Victorians understood, if you want to change your life, don’t just look for a clever cue. Commit to some larger global belief.  

96. Which of the following is a key element in the 19th-century character model? 

A. Passion.  

B. Action.   

C. Capability.   

D. Determination.  

97. The 19th-century model supposedly does not work because   ______.  

A. there were many other factors beyond one's control.  

B. it has worked unsatisfactorily most of the time.   

C. the comparison of free will to a dam is groundless.   

D. what one wishes to do should be considered carefully.   

98. What is the main implication of the research at Duke University? 

 A. Habit is key to one's behaviour.  

 B. One's behaviour is difficult to change.   

C. Both habit and will power are important.   

D. Habit has an unidentified structure.  

 

99. According to the new character model, personal behaviour could be altered through ______.  

 A. cues to stop all the former unconscious habits.  

B. cues to manipulate the habitual neural responses.   

C. techniques to devise different physical cues.   

D. techniques to supplement old routines.  

100. We learn from the passage that the new character model   ______.  

A. stresses the neural and psychological aspects of habit change.  

B. can bring about changes in one's life like what advertisers do.   

C. has been used to change behaviour successfully.   

D. deals better with emotional aspects of behaviour.  

 PART VI WRITING  

SECTION A COMPOSITION  45 MIN

Most of us, as students, are careful with our money in daily life: we collect all kinds of  coupons;

we look for group-buy deals if we eat out or travel; we don't buy clothes  unless in a sale. However, some people think that all this may not make us smart consumers.  What is your opinion?  

Write on ANSWER SHEET THREE a composition of about 200 words.  You are to write in three parts.  

In the first part, state specifically what your opinion is.  

In the second part, provide one or two reasons or your experience (s) to support your  opinion.  

In the last part, bring what you have written to a natural conclusion or make a  summary.  

You should supply an appropriate title for your composition.  

Marks will be awarded for content, organization, language and appropriateness.

 

SECTION B NOTE-WRITING [10 MIN]  Write on ANSWER SHEET THREE a note of about 50-60 words based on the following  situation:  

Your former high-school classmate (Mathew or Grace) is coming to visit you in the dorm.  But, you have got something urgent to do and thus cannot wait for him/her there. Leave a note  on the door, expressing your apology and telling him/her how to find you. You must make  clear where you are in the note.  

 Marks will be awarded for content, organization, language and appropriateness.   

附:【答案详解】 

PART 1 Dictation

What is a dream for?

One theory is that we dream to release the deep secret desires. We don't express these desires in real life because of the rules of polite society. Another theory is that dreams allow us to solve problems that we can't solve in real life. We go to sleep with the problem and wake up with the solution. This maybe a way to use our dreams rather than a purpose of dreaming. If you believe that your dreams are important, then analyzing them may help you to focus on the problem and help you to find the solution. The modern image is that the dreams are the brains' wave of cleaning up the computer's hard disk. Dreams organize the events of the day into folders and delete what is not needed. But we all know that very little of what we dream is concerned with what happened to us that day.

PART 2 LITENING COMPREHENSION

1-10 ABCAD ACAAD

11-20 DCADB BBDAC

21-30 DBADB CBBCA

PART 3 CLOZE

31-40 DACBD CBACB

41-50 DCADB BCDBB

PART 4 GRAMMAR & VOCABULARY

51. Facing the board of directors,he didn't  deny __________ breaking the agreement.  

A. him B. it C. his D.  its  

解析:本题考查动名词的逻辑主语。动名词可以有自己的逻辑(意义)主语,一般可以用名词所有格Toms,代词宾格如him(口语),书面语情况下一般用物主代词histheir。本题选C,但我还是要吐槽,因为根本就不需要多此一举添加his, he didn't  deny breaking the  

agreement完全正确,比原题要精简地道许多,从写作角度来看,his根本是多余的,当然纯粹考察语法的话选C 

 52. Xinchun returned from aboard a  different man. The italicized part functions as a(n) _______.  A. appositive(同位语) B. object C.  adverbial D. complement.  

解析:此题恐怕是最受争议的题目了,有人认为选A,有人认为选D。据目前来看,选D,具体讲解有待更新    

53. Which of the following is a compound word  (复合词)  

A. Nonsmoker B. Deadline C.Meaness D. Misfit   

解析:首先要知道什么是复合词。A compound is a word that consists of more  

than one free morpheme。所谓morpheme词素是最小的音义结合体,其最大的特点是不能再被分割为更小的音义结合体。而free  

morpheme是指能独立存在使用的词素,bound morpheme则是必须依附于其他单位的词素,比如前缀后缀属于bound  morpheme,如pre--ment,在单词shipment里,ship就是free  

morphemement则是bound。因此四个选项里只有deadline是由两个free morpheme构成(dead+line),其他三个都有Bound  

morpheme比如non,ness,mis-等。  

54. Which of the following sentences contains  subjunctive mood?  

A. Lucy insisted that her son get home before 5  o'clock?  

B. She used to drive to work, but now she takes  the city metro.  

C. Walk straight ahead, and don't turn till the  second traffic lights.  

D. Paul will cancel his flight if he cannot get  

his visa by Friday.  解析: subjunctive  

mood是指虚拟语气,四个选项里只有Ainsist属于虚拟语气用法,其中that从句里省略了should   

55.The-following determiners can be used  with both plural and uncountable nouns EXCEPT  A. more B. enough C.many  D. such  

解析:限定词的用法是近几年专四的热门考点,诸如somemanybothalleach等用法究竟如何还是希望大家多翻阅语法书和词典。这道题看似有点难度,也许你从来不知道哪个限定词可以接名词复数和不可数名词,但好在备选项里每个单词你都可以去接名词试试就知道了,比如more  

money,more trees; enough money, enough trees; many money?, many trees; such big  trees, such behavior.所以选C   

56. Which of the italicized parts indicates  CONTRAST?  

A. She opened the door and quietly  went in.  

B. Victoria likes music and Sam is  fond of sports.  

C. Think it over again and you'll get  an answer.  

D. He is somewhat arrogant, and I  don't like this.  

解析:首先要明白and不一定表示并列。AC里的and表示先后;D里的and表示递进;只有B里的and表示对比。    

57. Which of the following CANNOT be used as a  nominal substitute(名词替代词)  A. Much B. Neither C. One  D. Quarter   

解析:估计不少学生看到名词替代词这个术语,一开始都吓着了,因为压根没听说过,不过对做这道题目倒无大碍,所谓名词替代词顾名思义就是指能代替名词或名词短语,避免重复的词,英语里one,  

ones,(the)same等都是常见的。比如There are good films as well as bad ones.除此以外还有the kind,  the sort,比如Slang disappears quickly, especially the juvenile  

sort.还有一些不定代词等比如all, both, some, any enough, several, none, many,  much, (a) few, (a) little, the other, others, another, either,neither等,比如Can you get me some nails? I need some. I don’t want  any more food. I’ve had enough. 本题选D, quarter不是名词替代词。   

58. All the following sentences definitely  indicate future time EXCEPT  

A. Mother is to have tea with Aunt Betty at  four.  

B. The President is coming to the UN next  week,  

C. The school pupils will be home by  now.  

D. He is going to email me the necessary  information.  

解析:这道题目有点意思,既考查时态,又考查will的用法,A/B/D都是将来时的各种表达,唯独Cwill,看似是将来时标志,其实不然,因为和by  

now矛盾,这里will的用法是指you think something is true,表说话人自己的看法,比如That will be Tim coming  home now; Ask anyone and they will tell you the same thing. 本题选C   

59. Which of the following sentences is  grammatically INCORRECT?  A. Politics are the art or science of  government.  

B. Ten miles seems like a long walk to  me.  

C. Mumps is a kind of infectious  disease.  

D. All the furniture has arrived  undamaged.  

解析:本题考查主谓一致(agreement)。ten  

miles看作整体单数,mumps腮腺炎这个单词就是以s结尾的,不可数名词;furniture在英语里是不可数名词;A里的politics并不是指政治,而是指政治学(political  

science),是不可数名词,但这个用法主要是英式英语。此外,politics表示政治可以看作不可数名词,也可以看作复数,比如Politics have  

always interested me. 本题选A   

60. Which of in the following phrases indicates  a subject-predicate relationship?  A. The arrival of the tourists  B. The law of Newton  C.The occupation of the island  D. The plays of Oscar Wilde  

解析:传统语法和现代句法学关于什么是predicate本身就不一样,题目的传统语法predicate指谓语,是对主语动作或状态的陈述或说明,指出“做什么”“是什么”或“怎么样”。其次,说实话,我手上所有图书查不到相关语法点讲解subject-predicate  

relationship,谷歌后也没有什么结果,但我知道是选A,游客到来,只是我暂时找不出任何可靠来源来解释。   

61. Which of the following italicized parts  serves as an appositive?  A. He is not the man to draw  back.  

B.Tony hit back the urge to tell a  lie.  

 C. Larry has a large family to  support.  

D. There is really nothing to  fear.  

解析:又是同位语?!但是翻遍手上的各类参考书,都没有关于不定式to的同位语用法。四个选项的用法在语法书里并没有做进一步区分,都成为不定式的定语用法,修饰名词。不过还是可以看出不太一样的地方,A中名词和不定式是动宾结构the  

man draw backCD都是不定式和名词构成动宾to fear nothing, to support  family;而唯独B,urge(冲动)的内容就是指to tell a lie撒谎,故为同位语。   

62. Which of the following is NOT an imperative  sentence?  

A. Let me drive you home, shall I?  B. You will mind your own business.  C. Come and have dinner with us.  D. I wish you could stay behind.  

解析:花了点时间翻遍手头各类语法和用法图书,除了直接用动词打头,Do/Don'tYou/其他唤名+动词,  let类型外,均没有见到Dwish类型。   

63. If it _______ tomorrow, the match would be  put off.  

A. were to rain B. was to  rain C. was raining D. had rained  

解析:最经典的If条件虚拟句,说的是将来,应该选A.   

64. Which of the following sentences expresses  a fact?  

A. Mary and her son must be home by now.   

B. Careless reading must give poor  results.  

C. It's getting late, and I must leave  now.  

D. He must be working late at the  office.  

解析:很多人都以为选B,其实应该选CABD都是推测,当然了C里说的fact其实是指天色不早了。请注意B,不要以为有了must就是fact,情态动词再华丽还是表示情态,说话人的态度观点,B要改为Careless  reading gives poor results才是fact   

65. The following are all dynamic verbs(动态动词)  EXCEPT  

A. remain B. turn C. write D.  knock  

解析:看到专四考这个题目,我只能呵呵。动态动词是指能够延续持续动作的,比如hitrun等,相反的则是stative  verb静态动词,比如know.现在进行时是常见检验手段。你不能说I am knowing the answer,只能说I know the  answer,因此know是静态动词。同理,题目的A选项remain也是静态动词,其他都是动态动词。

66-70 CABDA 71-75 CBDAC 76-80 CADBA

PART 5 READING COMPREHENSION

81-85 CACDB

86-90 DACBC

91-95 BDDAC

96-100 DAACA

 PART 6 WRITING  

SECTION A COMPOSITION 

The advantages of being careful with money in daily life

Nowadays, there is a hot debate on whether people, including students should be careful with their money in their daily life since they try their best to save money. Some people think it is not good to do so because this may not make people smart consumers. But in my eyes, the advantages of being careful with money in daily life outweigh the disadvantages.

First, being careful with money is a virtue that can always benefit people. Being thrifty is of great help, especially when the economic status around the world is not very good and it becomes harder to earn money than before. A good habit of being careful with money will help many people go through this difficult period or any other difficult financial situation in their life. People with this awareness will think carefully before deciding to buy anything. For students, this awareness is also of great help since they mainly depend on their parents for financial supports. When they keep this awareness in their mind, they will definitely avoid wasting money.

Secondly, being careful with money is, in fact, a good way of practicing how to manage

one's money. In current society, it is necessary to learn something about finance. But theories are usually hard to learn. In contrast, most people can learn quickly by practicing. Being careful with money means an efficient use of money, which, in essence, equals to part of the job of a financial planner. Being careful with money can teach people to make best use of their money.

It is true that being too careful with money may have some disadvantages; for example, it may cause people to ignore other aspects of commodities. But overall this behavior is good and has its reality basis. Money-consciousness is always needed in the market-oriented economy.

SECTION B April 20

Dear Mathew,

I'm writing to show my apology for being unable to wait for you in my dorm. I'm really sorry but I have something urgent to do. You can come to Room 1003, No. 3 Teaching Building to find me. The map on the back of the note will help you. I'll wait for you there.

Yours, Tracy

TEST FOR ENGLISH MAJORS (2011)

-GRADE FOUR-

SECTION A CONVERSATIONS

In this section you will hear several conversations. Listen to the conversations carefully and then answer the questions that follow.

Questions 1 to 3 are based on the following conversation. At the end of the conversation, you will be given 15 seconds to answer the questions.

Now, listen to the conversation.

1. Which of the statements about the auto show is INCORRECT?

A. The show will have more stands this year.

B. The show will have more visitors this year.

C. The number of overseas visitors will be the same this year.

D. The number of exhibition days will be the same this year.

2. According to the conversation, the price for a stand would include

A. a catalogue.

B. a poster.

C. two desks.

D. four chairs.

3. During the conversation, the man seems to be more interested in

A. the size of the show.

B. the cost of the stand.

C. the basic furniture.

D. the visitor number.

Questions 4 to 7 are based on the following conversation. At the end of the conversation, you will be given 20 seconds to answer the questions.

Now, listen to the conversation.

4. What is Jim interested in?

A. Investing in the company’s product.

B. Buying a new educational computer.

C. Joining the computer company.

D. Reading the campaign plan.

5. The advertisements will be placed in all the following EXCEPT

A. family magazines.

B. Sunday newspapers.

C. morning television.

D. teenage magazines.

6. The advertisements are intended to be seen by

A. parents only.

B. children only.

C. school teachers.

D. whole families.

7. Linda sounds about the success of the campaign plan.

A. confident

B. hesitant

C. uncertain

D. delighted

Questions 8 to 10 are based on the following conversation. At the end of the conversation, you will be given 15 seconds to answer the questions.

Now, listen to the conversation.

8. John has worked in all the following places EXCEPT

A. a book shop.

B. a paper factory.

C. a school.

D. a fastfood restaurant.

9. From the conversation, we learn that John

A. has no interest.

B. has only one interest.

C. has two interests.

D. has quite a few interests.

10. Sue Green seems to be more interested in John’s experience of

A. working in a paper factory.

B. working in a youth club.

C. looking after his brother and sister.

D. looking after his young friends.

SECTION B PASSAGES

In this section, you will hear several passages. Listen to the passages carefully and then answer the questions that follow.

Questions 11 to 13 are based on the following passage. At the end of the passage, you will be given 15 seconds to answer the questions.

Now, listen to the passage.

11. According to the passage, the average age of the fleet of Delta/Northwest is

A. 10 years.

B. 14 years.

C. 16 years.

D. 20 years.

12. It can be learned from the passage that

A. air journeys are quite often far from comfortable.

B. air travellers usually enjoy luxury during flight.

C. air travellers usually enjoy upgraded conditions.

D. airlines always provide good in-flight entertainment.

13. Older planes can stay safe for more than 25 years because of the following EXCEPT

A. new engines.

B. strong materials.

C. proper maintenance.

D. new interior.

Questions 14 to 17 are based on the following passage. At the end of the passage, you will be given 20 seconds to answer the questions.

Now, listen to the passage.

14. Which job will involve supervising workers using dangerous equipment?

A. Assistant site manager.

B. Carpenter supervisor.

C. Assembly supervisor.

D. Automobile service station manager.

15. Who will also decide on the prices of products and services?

A. Airlines’ flight service manager.

B. Automobile service station manager.

C. Assistant site manager.

D. Carpenter supervisor.

16. Who may also do some of the work he supervises?

A. Assistant site manager.

B. Airlines’ flight service manager.

C. Assembly supervisor.

D. Carpenter supervisor.

17. Which job offers the highest salary?

A. Assistant site manager.

B. Automobile service station manager.

C. Carpenter supervisor.

D. Airlines’ flight service manager.

Questions 18 to 20 are based on the following passage. At the end of the passage, you will be given 15 seconds to answer the questions.

Now, listen to the passage.

18. Where were the two men filmed?

A. In the jewelry shop.

B. In the City Mall.

C. Near the lorry.

D. In the parking lot.

19. Witnesses saw the two men

A. leave the lorry together.

B. leave the lorry without hoods.

C. run back to the lorry separately.

D. run back to the lorry without hoods.

20. Which of the following details is CORRECT?

A. The lorry was originally painted white.

B. The lorry had no registration plates.

C. The shorter man was the passenger.

D. The taller man was the driver.

SECTION C NEWS BROADCAST

In this section, you will hear several news items. Listen to them carefully and then answer the questions that follow.

Questions 21 and 22 are based on the following news. At the end of the news item, you will be given 10 seconds to answer the questions.

Now, listen to the news.

21. What did NASA’s Constellation Programme originally plan to do?

A. To set up a moon colony by 2020.

B. To send astronauts again to the moon by 2020.

C. To continue the current shuttle missions till 2020.

D. To create more jobs for NASA till 2020.

22. NASA’s Constellation Programme would be cancelled mainly because

A. there were important space missions.

B. the space agency lacked funding for the programme.

C. the current shuttle missions would continue.

D. Congress failed to pass President Obama’s budget.

Questions 23 and 24 are based on the following news. At the end of the news item, you will be given 10 seconds to answer the questions.

Now, listen to the news.

23. The Russian cargo ship was sinking. What was the direct cause?

A. No explanation of the problem.

B. Long distance away from land.

C. Slow rescue efforts.

D. Severe weather.

24. Which of the following was NOT mentioned as a possible means of rescue?

A. Aircraft.

B. Tugboat.

C. Nearby cargo ship.

D. Vessels at the oil rig.

Questions 25 and 26 are based on the following news. At the end of the news item, you will be given 10 seconds to answer the questions.

Now, listen to the news.

25. Why did the singers meet in Hollywood?

A. To raise money for African humanitarian efforts.

B. To raise money for Haitian earthquake victims.

C. To sing in memory of Michael Jackson.

D. To make a recording of the original version of the song.

26. Which o the following details about the news is INCORRECT?

A. The organizer is one of the co-writers of the first song.

B. Singers will use the same recording studio as in 1985.

C. The recording session s expected to last long.

D. Stars from the original version will take part.

Question 27 and 28 are based on the following news. At the end of the news item, you will be given 10 seconds to answer the questions.

Now, listen to the news.

27. The additional 2 billion dollars will mainly be used to

A. upgrade its network capacity.

B. improve customer services.

C. develop new products.

D. market more iPhones.

28. Where does AT&T face difficulties in particular?

A. Nationwide.

B. Overseas.

C. In large cities.

D. In remote towns.

Questions 29 and 30 are based on the following news. At the end of the news item, you will be given 10 seconds to answer the questions.

Now, listen to the news.

29. Where is the presence of security patrols most evident?

A. Where matches take place.

B. Where protests may take place.

C. In downtown Vancouver.

D. Around the athletes’ village.

30. What is this news item mainly about?

A. Political protest during the Olympics.

B. Security operations during the Olympics.

C. Olympics’ security forces.

D. Security measures in buildings.

PART III CLOZE [15 MIN]

Decide which of the choices given below would best complete the passage if inserted in the corresponding blanks. Mark the best choice for each blank on Answer Sheet Two.

“Congratulations, Mr. Cooper. It’s a girl.”

Fatherhood is going to have a different meaning and (31) a different response from every man who hears these words. Some feel (32) when they receive the news, (33) others worry, wondering whether they will be good father. (34) there are some men who like children and may have had (35) experience with them, others do not particularly (36) children and spend little time with them. Many fathers and mothers have been planning and looking forward to children for some time. (37) other couples, pregnancy was an accident that both husband and wife have (38) willingly or unwillingly.

Whatever the (39) to the birth of a child, it is obvious the shift from the role of husband to (40) of a father is a difficult task. (41) , unfortunately, few attempts have been made to (42) fathers in this resocialization (43) . Although numerous books have been written about mothers, (44) recently has literature focused on the (45) of a father.

It is argued that the transition to the father's role, although difficult, is not (46) as great as the transition the wife must (47) to the mother's role. The mother's role seems to require a complete (48) in daily routine. (49) , the father’s role is less demanding and (50) .

31. A. bring down B. bring forth C. bring off D. bring in

32. A. emotional B. sentimental C. bewildered D. proud

33. A. while B. when C. if D. as

34. A. When B. If C. Although D. Yet

35. A. considerate B. considerable C. considering D. considered

36. A. care about B. care of C. care with D. care for

37. A. For B. Of C. From D. Upon

38. A. received B. taken C. accepted D. obtained

39. A. reply B. reaction C. readiness D. reality

40. A. what B. this C. one D. that

41. A. As a result B. For example C. Yet D. Also

42. A. educate B. cultivate C. inform D. convert

43. A. step B. process C. point D. time

44. A. / B. just C. quite D. only

45. A. role B. work C. career D. position

46. A. a little B. just C. nearly D. almost

47. A. take B. make C. carry D. accept

48. A. transformation B. realization C. socialization D. reception

49. A. In addition B. Above all C. Generally D. However

50. A. current B. immediate C. present D. quick

PART IV GRAMMAR & VOCABULARY [15 MIN]

There are thirty sentences in this section. Beneath each sentence there are four words or phrases marked A, B, C and D. Choose one word or phrase that best completes the sentence.

Mark your answers on Answer Sheet Two.

51. My uncle is quite worn out from years of hard work. He is no longer the man he was fifteen years ago.

A. which B. whom C. who D. that

52. Which of the following sentences is a COMMAND?

A. Beg your pardon.

B. Have a good time.

C. Never do that again!

D. What noise you are making!

53. Which of the following italicized phrases indicates purpose?

A. She said it for fun, but others took her seriously.

B. For all its effort, the team didn’t win the match.

C. Linda has worked for the firm for twenty years.

D. He set out for Beijing yesterday.

54. When you have finished with the book, don’t forget to return it to Tim, ?

A. do you B. will you C. don’t you D. won’t you

55. In phrases like freezing cold, burning hot, or soaking wet, the –ING participle is used

A. as a command.

B. as a condition.

C. for concession.

D. for emphasis.

56. Which of the following italicized phrases is INCORRECT?

A. The city is now ten times its original size.

B. I wish I had two times his strength.

C. The seller asked for double the usual price.

D. They come here four times every year.

57. It is not so much the language the cultural background that makes the book difficult to understand.

A. as B. nor C. but D. like

58. Which of the following italicized parts is used as an object?

A. What do you think has happened to her?

B. Who do you think the visiting professor is?

C. How much do you think he earns every month?

D. How quickly would you say he would come?

59. The additional work will take weeks.

A. the other B. another two C. other two D. the more

60. Which of the following italicized parts is a subject clause (主语从句)?

A. We are quite certain that we will get there in time.

B. He has to face the fact that there will be no pay rise this year.

C. She said that she had seen the man earlier that morning.

D. It is sheer luck that the miners are still alive after ten days.

61. It’s getting late. I’d rather you now.

A. left B. leave C. are leaving D. will leave

62. In the sentence “The manager interviewed Jim himself in the morning”, the italicized word is used to modify

A. the object.

B. the verb.

C. the subject.

D. the prepositional phrase.

63. There is no doubt the couple did the right thing in coming back home earlier than planned.

A. whether B. that C. why D. when

64. The sentence that expresses OFFER is

A. I’ll get some drinks. What’ll you have?

B. Does she need to book a ticket now?

C. May I know your name?

D. Can you return the book next week?

65. Which of the following italicized phrases indicates a subject-predicate relation (主谓关系)?

A. Mr Smith’s passport has been issued.

B. The visitor’s arrival was reported in the news.

C. John’s travel details have not been finalized.

D. The new bookstore sells children’s stories.

66. Our office has recently to a new computer system.

A. altered B. converted C. transformed D. modified

67. The crowd went as soon as the singer stepped onto the stage.

A. wild B. emotional C. uncontrolled D. unrestricted

68. Our school library is closed for repairs.

A. shortly B. quickly C. temporarily D. rapidly

69. John is up to his eyes in work at the moment. The underlined part means .

A. very excited B. very busy C. very tired D. very efficient

70. Victoria bumped into her brother quite by chance in the supermarket. The underlined word means .

A. risk B. opportunity C. possibility D. luck

71. “Look at those pretty girls’ skirts” is , because it is not clear whether the girls or the skirts are “pretty’.

A. ambiguous B. hidden C. indirect D. indistinct

72. House repairs, holidays, school fees and other have reduced his bank balance to almost nothing.

A. amount B. payment C. expenses D. figures

73. It was really of you to remember my birthday.

A. grateful B. thoughtful C. considerable D. generous

74. You can go to a travel agency and ask for a holiday .

A. introduction B. advertisement C. book D. brochure

75. The city government is building more roads to the increasing number of cars.

A. accommodate B. receive C. accept D. hold

76. They’ve lifted a two-year-long economic on the country.

A. enclosure B. restriction C. blockade D. prohibition

77. Everyone is surprised that she has fallen out with her boy friend. The underlined part means .

A. left B. quarreled C. attacked D. defeated

78. His plan is carefully prepared and full of details, so it is a very one.

A. elaborate B. refined C. ambitious D. complex

79. The girl’s voice was so low that we could hear her.

A. seldom B. almost C. only D. barely

80. She must have been pretty to fall for such an old trick.

A. interested B. gullible C. enthusiastic D. shrewd

PART V READING COMPREHENSION [25 MIN]

In this section there are four passages followed by questions or unfinished statements, each with four suggested answers marked A, B, C and D. Choose the one that you think is the best answer.

Mark your answers on Answer Sheet Two.

TEXT A

We have a crisis on our hands. You mean global warning? The world economy? No, the decline of reading. People are just not doing it anymore, especially the young. Who’s responsible? Actually, it’s more like, What is responsible? The Internet, of course, and everything that comes with it — Facebook, Twitter (微博). You can write your own list.

There’s been a warning about the imminent death of literate civilization for a long time. In the 20th century, first it was the movies, then radio, then television that seemed to spell doom for the written world. None did. Reading survived; in fact it not only survived, it has flourished. The world is more literate than ever before — there are more and more readers, and more and more books.

The fact that we often get our reading material online today is not something we should worry over. The electronic and digital revolution of the last two decades has arguably shown the way forward for reading and for writing. Take the arrival of e-book readers as an example. Devices like Kindle make reading more convenient and are a lot more environmentally friendly than the traditional paper book.

As technology makes new ways of writing possible, new ways of reading are possible. Interconnectivity allows for the possibility of a reading experience that was barely imaginable before. Where traditional books had to make do with photographs and illustrations, an e-book can provide readers with an unlimited number of links: to texts, pictures, and videos. In the future, the way people write novels, history, and philosophy will resemble nothing seen in the past.

On the other hand, there is the danger of trivialization. One Twitter group is offering its followers single-sentence-long “digests” of the great novels. War and Peace in a sentence? You must be joking. We should fear the fragmentation of reading. There is the danger that the high-speed connectivity of the Internet will reduce our attention span — that we will be incapable of reading anything of length or which requires deep concentration.

In such a fast-changing world, in which reality seems to be remade each day, we need the ability to focus and understand what is happening to us. This has always been the function of literature and we should be careful not to let it disappear. Our society needs to be able to imagine the possibility of someone utterly in tune with modern technology but able to make sense of a dynamic, confusing world.

In the 15th century, Johannes Guttenberg’s invention of the printing press in Europe had a huge impact on civilization. Once upon a time the physical book was a challenging thing. We should remember this before we assume that technology is out to destroy traditional culture.

81. Which of the following paragraphs briefly reviews the historical challenges for reading?

A. Paragraph One.

B. Paragraph Two.

C. Paragraph Three.

D. Paragraph Four.

82. The following are all cited as advantages of e-books EXCEPT

A. multimodal content.

B. environmental friendliness.

C. convenience for readers.

D. imaginative design.

83. Which of the following can best describe how the author feels toward single-sentence-long novels?

A. Ironic.

B. Worried.

C. Sarcastic.

D. Doubtful.

84. According to the passage, people need knowledge of modern technology and to survive in the fast-changing society.

A. good judgment

B. high sensitivity

C. good imagination

D. the ability to focus

85. What is the main idea of the passage?

A. Technology pushes the way forward for reading and writing.

B. Interconnectivity is a feature of new reading experience.

C. Technology is an opportunity and a challenge for traditional reading.

D. Technology offers a greater variety of reading practice.

TEXT B

I know when the snow melts and the first robins (知更鸟) come to call, when the laughter of children returns to the parks and playgrounds, something wonderful is about to happen.

Spring cleaning.

I’ll admit spring cleaning is a difficult notion for modern families to grasp. Today’s busy families hardly have time to load the dishwasher, much less clean the doormat. Asking the family to spend the weekend collecting winter dog piles from the melting snow in the backyard is like announcing there will be no more Wi-Fi. It interrupts the natural order.

“Honey, what say we spend the weekend beating the rugs, sorting through the boxes in the basement and painting our bedroom a nice lemony yellow?” I say.

“Can we at least wait until the NBA matches are over?” my husband answers.

But I tell my family, spring cleaning can’t wait. The temperature has risen just enough to melt snow but not enough for Little League practice to start. Some flowers are peeking out of the thawing ground, but there is no lawn to seed, nor garden to tend. Newly wakened from our winter’s hibernation (冬眠), yet still needing extra blankets at night, we open our windows to the first fresh air floating on the breeze and all of the natural world demanding “Awake and be clean!”

Biologists offer a theory about this primal impulse to clean out every drawer and closet in the house at spring’s first light, which has to do with melatonin, the sleepytime hormone (激素) our bodies produce when it’s dark. When spring’s light comes, the melatonin diminishes, and suddenly we are awakened to the dusty, virus-filled house we’ve been hibernating in for four months.

I tell my family about the science and psychology of a good healthy cleaning at spring’s arrival. I speak to them about life’s greatest rewards waiting in the removal of soap scum from the bathtub, which hasn’t been properly cleaned since the first snowfall.

“I’ll do it,” says the eldest child, a 21-year-old college student who lives at home.

“You will? Wow!” I exclaim.

Maybe after all these years, he’s finally grasped the concept. Maybe he’s expressing his rightful position as eldest child and role model. Or maybe he’s going to Florida for a break in a couple of weeks and he’s being nice to me who is the financial-aid officer.

No matter. Seeing my adult son willingly cleaning that dirty bathtub gives me hope for the future of his 12-year-old brother who, instead of working, is found to be sleeping in the seat of the window he is supposed to be cleaning.

“Awake and be clean!” I say.

86. According to the passage, …spring cleaning is difficult notion for modern families to grasp” means that spring cleaning

A. is no longer an easy practice to understand.

B. is no longer part of modern family life.

C. requires more family members to be involved.

D. calls for more complicated skills and knowledge.

87. Which of the following is LEAST likely to be included in family spring cleaning?

A. Beating the rugs.

B. Cleaning the window.

C. Restoring Wi-Fi services.

D. Cleaning the backyard.

88. Why does the author say “spring cleaning can’t wait”?

A. Because there will be more activities when it gets warmer.

B. Because the air is fresher and the breeze is lighter.

C. Because the whole family is full of energy at spring time.

D. Because the snow is melting and the ground is thawing.

89. Which of the following interpretations of the biologists’ theory about melatonin is INCORRECT?

A. The production of melatonin in our bodies varies at different times.

B. Melatonin is more likely to cause sleepiness in our bodies.

C. The reduction of melatonin will cause wakefulness in our bodies.

D. The amount of melatonin remains constant in our bodies.

90. Which of the following can best sum up the author’s overall reaction to her adult son’s positive response to spring cleaning?

A. Surprised and skeptical.

B. Elated and hesitant.

C. Relieved and optimistic.

D. Optimistic and hesitant.

TEXT C

These days lots of young Japanese do omiai, literally, “meet and look.” Many of them do so willingly. In today’s prosperous and increasingly conservative Japan, the traditional omiai kekkon, or arranged marriage, is thriving.

But there is a difference. In the original omiai, the young Japanese couldn’t reject the partner chosen by his parents and their middleman. After World War II, many Japanese abandoned the arranged marriage as part of their rush to adopt the more democratic ways of their American conquerors. The Western ren’ai kekkon, or love marriage, became popular; Japanese began picking their own mates by dating and falling in love.

But the Western way was often found wanting in an important respect: it didn’t necessarily produce a partner of the right economic, social, and educational qualifications. “Today’s young people are quite calculating,” says Chieko Akiyama, a social commentator.

What seems to be happening now is a repetition of a familiar process in the country’s history, the “Japanization” of an adopted foreign practice. The Western ideal of marrying for love is accommodated in a new omiai in which both parties are free to reject the match. “Omiai is evolving into a sort of stylized introduction,” Mrs. Akiyama says.

Many young Japanese now date in their early twenties, but with no thought of marriage. When they reach the age — in the middle twenties for women, the late twenties for men — they increasingly turn to omiai. Some studies suggest that as many as 40% of marriages each year are omiai kekkon. It’s hard to be sure, say those who study the matter, because many Japanese couples, when polled, describe their marriage as a love match even if it was arranged.

These days, doing omiai often means going to a computer matching service rather than to a nakodo. The nakodo of tradition was an old woman who knew all the kids in the neighbourhood and went around trying to pair them off by speaking to their parents; a successful match would bring her a wedding invitation and a gift of money. But Japanese today find it’s less awkward to reject a proposed partner if the nakodo is a computer.

Japan has about five hundred computer matching services. Some big companies, including Mitsubishi, run one for their employees. At a typical commercial service, an applicant pays $80 to $125 to have his or her personal data stored in the computer for two years and $200 or so more if a marriage results. The stored information includes some obvious items, like education and hobbies, and some not-so-obvious ones, like whether a person is the oldest child. (First sons, and to some extent first daughter, face an obligation of caring for elderly parents.)

91. According to the passage, today’s young Japanese prefer

A. a traditional arranged marriage.

B. a new type of arranged marriage.

C. a Western love marriage.

D. a more Westernized love marriage.

92. Which of the following statements is CORRECT?

A. A Western love marriage tends to miss some Japanese values.

B. Less attention is paid to the partner’s qualification in arranged marriages.

C. Young Japanese would often calculate their partner’s wealth.

D. A new arranged marriage is a repetition of the older type.

93. According to the passage, the figure 40% (Paragraph Five) is uncertain because

A. there has been a big increase in the number of arranged marriages.

B. Western love marriage still remains popular among young Japanese.

C. young Japanese start dating very early in their life in a Western tradition.

D. the tendency for arranged marriages could be stronger than is indicated.

94. One of the big differences between a traditional nakodo and its contemporary version lies in the way

A. wedding gifts are presented.

B. a proposed partner is refused.

C. formalities are arranged.

D. the middleman/woman is chosen.

95. What is the purpose of the last paragraph?

A. To tell the differences between an old and modern nakodo.

B. To provide some examples for the traditional nakodo.

C. To offer more details of the computerized nakodo.

D. To sum up the main ideas and provide a conclusion.

TEXT D

Cordia Harrington was tired of standing up all day and smelling like French fries at night. She owned and operated three McDonald’s shops in Illinois, but as a divorced mother of three boys, she yearned for a business that would provide for her children and let her spend more time with them.

Her lucky moment came, strangely enough, after she was nominated in 1992 to be on the McDonald’s bun committee. “The company picked me up in a corporate jet to see bakeries around the world,” she recalls. “Every time I went to a meeting, I love it. This was global!”

The experience opened her eyes to business possibilities. When McDonald’s decided it wanted a new bun supplier, Harrington became determined to win the contract, even though she had no experience running a bakery.

Harrington studied the bakery business and made sure she was never off executives’ radar. “If you have a dream, you can’t wait for people to call you,” she says. “So I’d visit a mill and send them photos of myself in a baker’s hat and jacket, holding a sign that say ‘I want to be your baker.’ ” After four years and 32 interviews, her persistence paid off.

Harrington sealed the deal with a handshake, sold her shops, and borrowed $13.5 million. She was ready to build the fastest, most automated bakery in the world.

The Tennessee Bun Company opened ahead of schedule in 1997, in time for a slump in U.S. fast-food sales for McDonald’s. Before Harrington knew it, she was down to her last $20,000, not enough to cover payroll. And her agreement with McDonald’s required that she sell exclusively to the company. “I cried myself to sleep many nights,” she recalls. “I really did think, I am going to go bankrupt.”

But Harrington worked out an agreement to supply Pepperidge Farm as well. “McDonald’s could see a benefit if our production went up and prices went down, and no benefit if we went out of business,” she says. “That deal saved us.”

Over the next eight years, Harrington branched out even more: She started her own trucking business, added a cold-storage company, and now has three bakeries producing fresh buns and frozen dough — all now known as the Bun Companies. Speed is still a priority: It takes 11 people at the main bakery to turn out 60,000 buns an hour for clients across 40 states, South America, and the Caribbean.

Grateful for the breaks she’s had, Harrington is passionate about providing opportunities to all 230 employees. “Financial success is the most fun when you can give it away,” she says.

The current economy is challenging. Some of her clients’ sales have declined, but she’s found new clients and improved efficiencies to help sustain the company’s double-digit growth.

Cordia Harrington doesn’t have to stand on her feet all day anymore. Two of her three sons now work for her. And she’s remarried — her husband, Tom, is now her CFO.

“This is more than a job,” says Harrington. “It’s a mission. I’m always thinking. How can we best serve our employees? If we support them, they’ll do their best to look after our clients. That’s how it works here.”

96. According to the passage, which of the following was most significant in her early career?

A. Her nomination on the McDonald’s bun committee.

B. Her travel and the visits to bakeries around the world.

C. A business contract with local bun suppliers.

D. The interviews and experience in running a bakery.

97. “Harrington…made sure she was never off executives’ radar” (Paragraph Four) means that she

A. herself wanted to be a company executive.

B. meant to hire executives to run the business.

C. meant to keep her management knowledge and skills.

D. focused on the management of the bakery business.

98. How did she survive the crisis at the start of her bakery business?

A. By supplying buns for another company.

B. By opening her bun company ahead of schedule.

C. By keeping supplies up for McDonald’s.

D. By making a new agreement with McDonald’s.

99. Which of the following statements is INCORRECT in describing her current business?

A. It is fast growing.

B. It is diversified.

C. Its clients are all local.

D. It is more efficient.

100. According to the passage, which of the following is fundamental to Harrington’s success?

A. Efficiency and love for the family.

B. Perseverance and concern for employees.

C. Business expansion and family support.

D. Opportunities and speed.

PART VI WRITING [45 MIN]

SECTION A COMPOSITION [35 MIN]

Recently government agencies in some big cities have been studying the possibility of putting a “pollution tax” on private cars. The amount of tax private car owners would have to pay would depend on the emission levels, i.e. engine or vehicle size. This has caused quite a stir among the public. Some regard it as an effective way to control the number of cars and reduce pollution in the city. But others don’t think so. What is your opinion?

Write on ANSWER SHEET THREE a composition of about 200 words on the following topic:

Should Private Car Owners be Taxed for Pollution?

You are to write in three parts.

In the first part, state specifically what your opinion is.

In the second part, provide one or two reasons to support your opinion.

In the last part, bring what you have written to a natural conclusion or make a summary.

SECTION B NOTE-WRITING [10 MIN]

Write on ANSWER SHEET THREE a note of about 50-60 words based on the following situation:

During the summer vacation you would like to invite your best friend to your home town. Write a note to him/her, extending your invitation and telling him/her how to get to your home town.

Marks will be awarded for content, organization, language and appropriateness.

2010年专四真题

In this section you will hear several conversations. Listen to the conversations carefully and

then answer the questions that follow.

Questions 1 to 3 are based on the following conversation. At the end of the conversation, you will be given 15 seconds to answer the questions. Now, listen to the conversation.

1. The following details have been checked during the conversation EXCEPT

A. number of travelers. B. number of tour days.

C. flight details. D. room services.

2. What is included in the price?

A. Air tickets and local transport. B. Local transport and meals.

C. Air tickets, local transport and breakfast.

D. Air tickets, local transport and all meals.

3. Which of the following statements is CORRECT?

A. The traveler is reluctant to buy travel insurance.

B. The traveler is ready to buy travel insurance.

C. The traveler doesn't have to buy travel insurance.

D. Travel insurance is not mentioned in the conversation.

Questions 4 to 7 are based on the following conversation. At the end of'the conversation, you will be given 20 seconds to answer the questions. Now, listen to the conversation.

4. Which of the following details is CORRECT?

A. Mark knows the exact number of airport buses.

B. Mark knows the exact number of delegates' spouse.

C. Mark doesn't know the exact number of delegates yet.

D. Mark doesn't know the number of guest speakers.

5. What does Linda want to know?

A. The arrival time of guest speakers.

B. The departure time of guest speakers.

C. The type of transport for guest speakers.

D. The number of guest speakers.

6. How many performances have been planned tbr the conference?

A. One. B. Two. C. Three. D. Not mentioned.

7. Who will pay for the piano performance?

A. Pan-Pacific Tours. B. Johnson & Sons Events.

C. Conference delegates. D. An airline company.

Questions 8 to 10 are based on the following conversation. At the end of'the conversation,

you will be given 15 seconds to answer the questions. Now listen to the conversation.

8. What is NOT missing in Mary's briefcase?

A. Her cheque book. B. Her papers for work.

C. Her laptop. D. Her appointment book.

9. Where was Mary the whole morning?

A. At the police station. B. At a meeting.

C. In her client's office. D. In the restaurant.

10. Why was Mary sure that the briefcase was hers in the end?

A. The papers inside had the company's name.

B. The briefcase was found in the restaurant.

C. The restaurant manager telephoned James.

D. The cheque book inside bore her name.

SECTION B PASSAGES

In this section, you will hear several passages. Listen to the passages carefully and then answer the questions that follow.

Questions 11 to 13 are based on the following passage. At the end of the passage, yott will be given 15 seconds to answer the questions. Now, listen to the passage.

11. We learn from the passage that about two-thirds of the courses are taught through

A. the School of Design and Visual Arts. B. the School of Social Work.

C. the School of Business. D. the Arts and Sciences program.

12.What is the cost of undergraduate tuition?

A. Twenty thousand dollars. B. Thirty thousand dollars.

C. Twenty-seven thousand dollars. D. Thirty-eight thousand dollars.

13.International students can receive all the following types of financial assistance EXCEPT

A. federal loans. B. private loans.

C. scholarships. D. monthly payment plans.

ass=MsoNormal> Questions 14 to 17 are based on the following passage. At the end of the passage, you will be

given 20 seconds to answer the questions. Now, listen to the passage.

14. According to the passage, mothers in ____ spend more time looking after children.

A. France B. America C. Denmark D. Australia

15. Which of the following activities would Australian fathers traditionally participate in?

A. Feeding and playing with children.

B. Feeding and bathing children.

C. Taking children to the park and to school.

D. Taking children to watch sports events.

16. According to the study, the "new man" likes to

A. spend more time at work. B. spend more time with children.

C. spend time drinking after work. D. spend time on his computer.

17.It is suggested in the passage that the "new man" might be less acceptable in

A. France. B. Britain. C. Australia. D. Denmark.

Questions 18 to 20 are based on the following passage. At the end of the passage, you will be given 15 seconds to answer the questions. Now, listen to the passage.

18.The services of the new partnership are provided mainly to

A. mothers of infected babies. B. infected children and women.

C. infected children in cities. D. infected women in cities.

19.Which of the following details about Family Health International is INCORRECT?

A. It is a nonprofit organization. B. It provides public health services.

C. It carries out research on public health. D. It has worked in five countries till now.

20.The example of Cambodia mainly shows

A. the importance of government support.

B. the importance of public education efforts.

C. the progress the country has made so far.

D. the methods used to fight AIDS.

SECTION C NEWS BROADCAST

In this section, you will hear several news items. Listen to them carefully and then answer the

questions that follow.

Questions 21 and 22 are based on the following news. At the end of the news item, you will be given 10 seconds to answer the questions. Now. listen to the news.

21. According to the news, the victim was

A. a 17-year-old girl. B. a 15-year-old boy.

C. a 23-year-old woman. D. an l 8-year-old man.

22.We learn from the news that the suspects were arrested

A. one month later. B. two months later.

C. immediately. D. two weeks later.

Questions 23 and 24 are based on the following news. At the end of the news item, you will be

given 10 seconds to answer the questions. Now, listen to the news.

23.The Iraqi par

liament can vote on the security agreement only after

A. all parties have agreed on it. B. the US troops have pulled out.

C. the cabinet has reviewed it. D. the lawmakers have returned from Mecca.

24.According to the news, the US troops are expected to completely pull out by

A. mid-2009. B. the end of 2009.

C. mid-2011. D. the end of 2011.

Questions 25 and 26 are based on the following news. At the end of the news item, you will be

given 10 seconds to answer the questions. Now, listen to the news.

25.The following are involved in the operations to rescue the children in Honduras EXCEPT

A. the police. B. the district attorney.

C. the prison authorities. D. Institute of Childhood and Family.

26. What punishment would parents face if they allowed their children to beg?

A. To be imprisoned and fined. B. To have their children taken away.

C. To be handed over to the authorities. D. None.

Question 27 is based on the following news. At the end of the news item. you will be given 5

seconds to answer the question. Now, listen to the news.

27.What is the news item about?

A. Coastlines in Italy. B. Public use of the beach.

C. Swimming and bathing. D. Private bathing clubs.

Question 28 is based on the following news. At the end of the news item, you will be given 5

seconds to answer the question. Now, listen to the news.

28.Which of the following is NOT mentioned in the news?

A. The airport was shut down for Friday.

B. There was a road accident involving two buses.

C. Local shops were closed earlier than usual.

D. Bus service was stopped for Friday.

Questions 29 and 30 are based on the following news. At the end of the news item, you will be

given 10 seconds to answer the questions. Now, listen to the news.

29.How many people were rescued from the apartment building?

A. 17. B. 24. C. 21. D. 41.

30.Which of the following details in the news is CORRECT?

A. The rescue operation involved many people.

B. The cause of the explosions has been determined.

C. Rescue efforts were stopped on Thursday.

D. The explosions didn't destroy the building.

PART III CLOZE [15 MIN]

Decide which of the choices given below would best complete the passage it" inserted in the

corresponding blanks. Mark the best choice for each blank on ANSWER SHEET TWO.

 

How men first learned to i nvent words is unknown; (31)____, the origin of language is a mystery. All we really know is that men, unlike animals, somehow invented certain (32)____ to express thoughts and feelings, actions and things, (33)____ they could communicate with each other; and that later they agreed (34)____ certain signs, called letters, which could be (35)____ to represent those sounds, and which could be (36)_____. Those sounds, whether spoken, (37)_____ written in letters, we call words.

The power of words, then, lies in their (38)____ the things they bring up before our minds. Words become (39)____ with meaning for us by experience; (40)._____ the longer we live, the more certain words (41)_____ to us the happy and sad events of our past: and the more we (42)____, the more the number of words that mean something to us (43)____

Great writers are those who not only have great thoughts but also express these thoughts in words which appeal (44)____ to our minds and emotions. This (45)._____ and telling use of words is what we call (46)____ style. Above all, the real poet is a master of (47)____. He can convey his meaning in words which sing like music, and which (48)_____ their position and association can (49)____ men to tears. We should, therefore, learn to choose our words carefully and use them accurately, or they will (50)____ our speech or writing silly and vulgar.

 

(31) A. in addition B. in other words C. in a word D. in summary

(32) A. sounds B. gestures C. signs D. movements

(33) A. such that B. as that C. so that D. in that

(34) A. in B. with C. of D. upon

(35) A. spelt B. combined C. written D copied

(36) A. written down B. handed down C. remembered D. observed

(37) A. and B. yet C. also D. or

(38) A. functions B. associations C. roles D. links

(39) A. filled B. full C. live D. active

(40) A. but B. or C. yet D. and

(41 ) A. reappear B. recall C. remember D. recollect

(42) A. read and think B. read and recall C. read and learn D. read and recite

(43) A. raises B. increases C. improves D. emerges

(44) A. intensively B. extensively C. broadly D. powerfully

(45) A. charming B. academic C. conventional D. common

(46) A. written B. spoken C. literary D. dramatic

(47) A. signs B. words C. style D. sound

(48) A. in B. on C. over&n bsp;    D. by

(49) A. move B. engage C. make D. force

(50) A. transform B. change C. make D. convert

PART IV GRAMMAR & VOCABULARY [15 MIN]

There are thirty sentences in this section. Beneath each sentence there are four words or phrases marked A, B, C and D. Choose one word or phrase that best completes the sentence. Mark your answers on ANSWER SHEET TWO.

51. Which of the following italicized phrases indicates CAUSE?

A. Why don't you do it for the sake of your friends?

B. I wish I could write as well as you.

C. For all his efforts, he didn't get an A.

D. Her eyes were red from excessive reading.

52. Nancy's gone to work but her car's still there. She ____ by bus.

A. must have gone B. should have gone

C. ought to have gone D. could have gone

53. He feels that he is not yet ____ to travel abroad.

A. too strong B. enough strong C. so strong D. strong enough

54. After____ seemed an endless wait, it was his turn to enter the personnel manager's office.

A. that B. it C. what D. there

55. Fool ____ Jerry is, he could not have done such a thing.

A. who B. as C. like D. that

56. Which of the following sentences is INCORRECT?

A. They each have two tickets.

B. They cost twenty yuan each.

C. Each they have bought the same book.

D. They were given two magazines each.

57. She seldom goes to the theatre, _____?

A. doesn't she B. does she C. would she D. wouldn't she

58. Dr Johnson is head of the department, ____ an expert in translation.

A. or B. either C. but D. and

59. When one has good health, _____ should feel fortunate.

A. you B. they C. he D. we

60. It is necessary that he ____ the assignment without delay.

A. hand in B. hands in C. must hand in D. has to hand in

61. In the sentence "It's no use waiting for her", the italicized phrase is)____.

A. the object B. an adverbial C. a complement D. the subject

62. Which of the following sentences is INCORRECT?

A. All his lectures are very interesting.

B. Half their savings were gone.

C. Many his friends came to the party.

D. Both his sisters are nurses.

63. Which of the following sentences has an object complement?

A. The directors appointed John manager.

B. I gave Mary a Christmas present.

C. You have done Peter a favour.

D. She is teaching children English.

64. Which of the following words can NOT be used to complete "We've seen the film ____"?

A. before B. recently C. lately D. yet

65. _____ should not become a serious disadvantage in life and work.

A. To be not tall B. Not being tall

C. Being not tall D. Not to be tall

66. Due to personality _____, the two colleagues never got on well in work.

A. contradiction B. conflict C. confrontation D. competition

67. During the summer vacation, kids are often seen hanging _____ in the streets.

A. about B. on C. over D. out

68. There were 150 ____ at the international conference this summer.

A. spectators B. viewers C. participants D. onlookers

69. School started on a ____ cold day in February.

A. severe B. bitter C. such D. frozen

70. In the face of unexpected difficulties, he demonstrated a talent for quick, ____ action.

A. determining B. defensive C. demanding D. decisive

71. The team has been working overtime on the research project ____.

A. lately B.just now C. late D. long ago

72. Because of the economic crisis, industrial output in the region remained

A. motionless       B. inactive C. stagnant D. immobile

73. The police had difficulty in ____ the fans fi'om rushing on to the stage to take photos with the singer.

A. limiting B. restraining C. confining D. restricting

74. Joan is in the dorm, putting the final ____ to her speech.

A. details B. remarks C. comments D. touches

75. His_____ in gambling has eventually brought about his ruin.

A. indulgence B. habit C. action D. engagement

76. The teacher told the students to stay in the classroom and they did _____.

A. absolutely B. accidentally C. accordingly D. accurately

77. You can actually see the deer at close range while driving through that area. The italicized

phrase means _____.

A. clearly B. very near C. quickly D. very hard

78. He listened hard but still couldn't what they were talking about.

A. make over B. make up C. make upon D. make out

79. For the advertised position, the company offers a(n) salary and benefits package.

A. generous B. plentiful C. abundant D. sufficient

80. As there was no road, the travelers ____ up a rocky slope on their way back.

A. ran B. hurried C. scrambled D. crawled

PART V READING COMPREHENSION [25 MIN]

In this section there are four passages followed by questions or unfinished statements, each with four suggested answers marked A, B, C and D. Choose the one that you think is the best answer. Mark your answers on ANSWER SHEET TWO.

TEXT A

What is the nature of the scientific attitude, the attitude of the man or woman who studies and applies physics, biology, chemistry, geology, engineering, medicine or any other science? We all know that science plays an important role in the societies in which we live. Many people believe, however, that our progress depends on two different aspects of science. The first of these is the application of the machines, products and systems of applied knowledge that scientists and technologists develop. Through technology, science improves the structure of society and helps man to gain increasing control over his environment.

The second aspect is the application by all members of society of the special methods of thought and action that scientists use in their work.

What are these special methods of thinking and acting? First of all, it seems that a successful scientist is full of curiosity - he wants to find out how and why the universe works. He usually directs his attention towards problems which he notices have no satisfactory explanation, and his curiosity makes him look for underlying relationships even if the data available seem to be unconnected. Moreover, he thinks he can improve the existing conditions and enjoys trying to solve the problems which this involves.

He is a good observer, accurate, patient and objective and applies logical thought to the observations he makes. He utilizes the facts he observes to the fullest extent. For example, trained observers obtain a very large amoun

t of information about a star mainly from the accurate analysis of the simple lines that appear in a spectrum.

He is skeptical - he does not accept statements which are not based on the most complete evidence available - and therefore rejects authority as the sole basis for truth. Scientists always check statements and make experiments carefully and objectively to verify them.

Furthermore, he is not only critical of the work of others, but also of his own, since he knows that man is the least reliable of scientific instruments and that a number of factors tend to disturb objective investigation.

Lastly, he is highly imaginative since he often has to look for relationships in data which are not only complex but also frequently incomplete. Furthermore, he needs imagination if he wants to make hypotheses of how processes work and how events take place.

These seem to be some of the ways in which a successful scientist or technologist thinks and acts.

81. Many people believe that science helps society to progress through

A. applied knowledge.

B. more than one aspect.

C. technology only.

D. the use of machines.

82. Which of the following statements is INCORRECT about curiosity?

A. It gives the scientist confidence and pleasure in work.

B. It gives rise to interest in problems that are unexplained.

C. It leads to efforts to investigate potential connections.

D. It encourages the scientist to look for new ways of acting.

83. According to the passage, a successful scientist would not

A. easily believe in unchecked statements.

B. easily criticize others' research work.

C. always use his imagination in work.

D. always use evidence from observation.

84. What does the passage mainly discuss?

A. Application of technology.

B. Progress in modem society.

C. Scientists' ways of thinking and acting.

D. How to become a successful scientist.

85. What is the author's attitude towards the topic?

A. Critical. B. Objective.

C. Biased. D. Unclear.

TEXT B

Over the past several decades, the U.S., Canada, and Europe have received a great deal of media and even research attention over unusual phenomena and unsolved mysteries. These include UFOs as well as sightings and encounters with "nonhuman creatures" such as Bigfoot and the Loch Ness monster. Only recently has Latin America begun to receive some attention as well. Although the mysteries of the Aztec, Mayan, and Inca civilizations have been known for centuries, now the public is also becoming aware of unusual, paranormal phenomena in countries such as Peru.

The Nazca "lines" of Peru were discovered in the 1930s. These lines are deeply carved into a flat, stony plain, and form about 300 intricate pictures of animals such as birds, a monkey, and a lizard. Seen at ground level, the designs are a jumbled senseless mess. The images are so large that they can only be viewed at a height of 1,000 feet - meaning from an aircraft. Yet there were no aircraft in 300 B.C., when it is judged the designs were made. Nor were there then, or are there now, any nearby mountain ranges from which to view them. So how and why did the native people of Nazca create these marvelous designs? One answer appeared in 1969, when the German researcher and writer Erich von Daniken proposed that the lines were drawn by extraterrestrials as runways for their aircraft. The scientific community did not take long to scoffat and abandon von Daniken's theory. Over the years several other theories have been put forth, but none has been accepted by the scientific community.

Today there is a new and heightened interest in the Nazca lines. It is a direct result of the creation of the Internet. Currently there a

re over 60 sites dedicated to this mystery from Latin America's past, and even respected scientists have joined the discussion through e-mail and chat rooms.

Will the Internet help explain these unsolved mysteries? Perhaps it is a step in the right direction.

86. Which of the following statements is INCORRECT?

A. Latin America has long received attention for unusual phenomena.

B. Public attention is now directed towards countries like Peru.

C. Public interest usually focuses on North America and Europe.

D. Some ancient civilizations have unsolved mysteries.

87. According to the passage, the Nazca lines were found

A. in mountains. B. in stones. C. on animals. D. on a plain.

88. We can infer from the passage that the higher the lines are seen, the ____ the images they present.

A. smaller B. larger C. clearer D. brighter

89. There has been increasing interest in the Nazca lines mainly because of

A. the participation of scientists. B. the emergence of the lnternet.

C. the birth of new theories. D. the interest in the Internet.

90. The author is ____ about the role of the lnternet in solving mysteries.

A. cautious B. pessimistic C. uncertain D. optimistic

TEXT C

Graduation speeches are a bit like wedding toasts. A few are memorable. The rest tend to trigger such thoughts as, "Why did I wear such uncomfortable shoes?"

But graduation speeches are less about the message than the messenger. Every year a few colleges and universities in the US attract attention because they've managed to book high-profile speakers. And, every year, the media report some of these speakers' wise remarks.

Last month, the following words of wisdom were spread:

"You really haven't completed the circle of success unless you can help somebody else move forward." (Oprah Winfrey, Duke University).

"There is no way to stop change; change will come. Go out and give us a future worthy of the world we all wish to create together." (Hillary Clinton, New York University).

"'This really is your moment. History is yours to bend." (Joe Biden, Wake Forest University).

Of course, the real "get" of the graduation season was first lady Michelle Obama's appearance at the University of California, Merced. "Remember that you are blessed," she told the class of 2009, "Remember that in exchange for those blessings, you must give something back... As advocate and activist Marian Wright Edelman says, 'Service is the rent we pay for living ... it is the true measure, the only measure of success'."

Calls to service have a long, rich tradition in these speeches. However, it is possible for a graduation speech to go beyond cliche and say something truly compelling. The late writer David Foster Wallace's 2005 graduation speech at Kenyon College in Ohio talked about how to truly care about other people. It gained something of a cult after it was widely circulated on the Internet. Apple Computer CEO Steve Jobs' address at Stanford University that year, in which he talked about death, is also considered one of the best in recent memory.

But when you're sitting in the hot sun, fidgety and freaked out, do you really want to be lectured about

the big stuff?. Isn't that like trying to maintain a smile at your wedding reception while some relative gives a toast that amounts to "marriage is hard work"? You know he's right; you just don't want to think about it at that particular moment. In fact, as is the case in many major life moments, you can't really manage to think beyond the blisters your new shoes are causing.

That may seem anticlimactic. But it also gets to the heart of one of life's greatest, saddest truths: that our most "memorable" occasions may elicit the fewest memories. It's probably not something most graduation speakers would say, but it's one of the first lessons of growing up.

91. According to the passage, most graduation speeches tend to recall ____ memories.

A. great B. trivial C. unforgettable D. unimaginative

92. "But graduation speeches are less about the message than the messenger" is explained

A. in the final paragraph. B. in the last but one paragraph.

C. in the first paragraph. D. in the same paragraph.

93. The graduation speeches mentioned in the passage are related to the following themes EXCEPT

A. death. B. success. C. service. D. generosity.

94. It is implied in the passage that at great moments people fail to

A. remain clear-headed. B. keep good manners.

C. remember others' words. D. recollect specific details.

95. What is "one of the first lessons of growing up"?

A. Attending a graduation ceremony.

B. Listening to graduation speeches.

C. Forgetting details of memorable events.

D. Meeting high-profile graduation speakers.

TEXT D

Cultural rules determine every aspect of food consumption. Who eats together defines social units. For example, in some societies, the nuclear family is the unit that regularly eats together. The anthropologist Mary Douglas has pointed out that, for the English, the kind of meal and the kind of food that is served relate to the kinds of social links between people who are eating together. She distinguishes between regular meals, Sunday meals when relatives may come, and cocktail parties for acquaintances. The food served symbolizes the occasion and reflects who is present. For example, only snacks are served at a cocktail party. It would be inappropriate to serve a steak or hamburgers. The distinctions among cocktails, regular meals, and special dinners mark the social boundaries between those guests who are invited for drinks, those who are invited to dinner, and those who come to a family meal. In this example, the type of food symbolizes the category of guest and with whom it is eaten.

In some New Guinea societies, the nuclear family is not the unit that eats together. The men take their meals in a men's house, separately from their wives and children. Women prepare and eat their food in their own houses and take the husband's portion to the men's house. The women eat with their children in their own houses. This pattern is also widespread among Near Eastern societies.

Eating is a metaphor that is sometimes used to signify marriage. In many New Guinea societies, like that of the Lesu on the island of New Ireland in the Pacific and that of the Trobriand Islanders, marriage is symbolized by the couple's eating together for the first time. Eating symbolizes their new status as a married couple. In U.S. society, it is just the reverse. A couple may go out to dinner on a first date.

Other cultural rules have to do with taboos against eating certain things. In some societies, members of a clan, a type of kin (family) group, are not allowed to eat the animal or bird that is their totemic ancestor. Since they believe themselves to be descended from that ancestor, it would be like eating that ancestor or eating themselves.

There is also an association between food prohibitions and rank, which is found in its most extreme form in the caste system of India. A caste system consists of ranked groups, each with a different economic specialization. In India, there is an association between caste and the idea of pollution. Members of highly ranked groups can be polluted by coming into contact with the bodily secretions, particularly saliva, of individuals of lower-ranked castes. Because of the fear of pollution, Brahmans and other high-ranked individuals will not share food with, no

96. According to the passage, the English make clear distinctions between

A. people who eat together.

B. the kinds of food served.

C. snacks and hamburgers.

D. family members and guests.

97. According to the passage, who will NOT eat together?

A. The English.

B. Americans on their first date.

C. Men and women in Near Eastern societies.

D. Newly-weds on the island of New Ireland.

98. According to the passage, eating together indicates all the following EXCEPT

A. the type of food. B. social relations.

C. marital status. D. family ties.

99. The last paragraph suggests that in India ____ decides how people eat.

A. pollution B. food C. culture D. social status

100. Which of the following can best serve as the topic of the passage?

A. Different kinds of food in the world.

B. Relations between food and social units.

C. Symbolic meanings of food consumption.

D. Culture and manners of eating.

PART VI WRITING [45 MIN]

TEST FOR ENGLISH MAJORS (2009)

-GRADE FOUR-

TIME LIMIT: 135 MIN

SECTION A CONVERSATIONS

In this section you will hear several conversations. Listen to the conversations carefully and then answer the questions that follow.

Questions 1 to 3 are based on the following conversation. At the end of the conversation, you will be given 15 seconds to answer the questions. Now, listen to the conversation.

1. Mark is unhappy because of

A. his Chemistry homework. B. a girl in his class.

C. Linda's words. D. Friday night's party.

2. Which of the following is CORRECT?

A. Linda is Jane's friend. B. Mark is Jane's boyfriend.

C. John is Jane's boyfriend. D. Mark and John are good friends.

3. Did Mark eventually take Linda's advice?

A. No. B. Partly. C. Completely. D. Not mentioned.

Questions 4 to 7 are based on the following conversation. At the end of the conversation, you will be given 20 seconds to answer the questions. Now, listen to the conversation.

4. About the scratch on the product, the shop assistant thinks that

A. the customer made it himself. B. there was definitely not one then.

C. the customer should have checked. D. the customer was making trouble.

5. The customer was ______ when told he might not have worn the headphones properly.

A. annoyed B. surprised C. indifferent D. worried

6. How many complaints did the customer make about the product altogether?

A. Five. B. Four. C. Three. D. Two.

7. The shop could exchange the product if the customer

A. makes no more complaints. B. can produce the receipt.

C. is still unhappy with it. D. brings it back within a week.

Questions 8 to 10 are based on the following conversation. At the end of the conversation, you will be given 15 seconds to answer the questions. Now, listen to the conversation.

8. Joe Smith telephoned Victoria for

A. the menu. B. the place. C. the reception. D. the campaign.

9. When will the lunch be held?

A. Friday next week. B. Thursday next week.

C. April 30th. D. This week.

10. All the following information is new to Victoria EXCEPT

A. how many people to attend it. B. why to hold it.

C. where to hold it. D. what to cook.

SECTION B PASSAGES

In this section, you will hear several passages. Listen to the passages carefully and then

answer the questions that follow.

Questions 11 to 13 are based on the following passage. ,4t the end of the passage, you will be given 15 seconds to answer the questions. Now, listen to the passage.

11. People choose London for post-Christmas shopping because

A. shops open early in the morning.

B. shops stay open for longer hours.

C. they can buy really cheap things.

D. they can shop with their friends.

12. We learn from the passage that

A. people are very keen on sales.

B. post-Christmas sales start at 3:30am.

C. post-Christmas sales last for a day.

D. sales include only a few items.

13. Which of the following is NOT mentioned in the passage?

A. Some people buy Christmas presents in the sales.

B. Some people shop online during the sales.

C. Some people buy presents for next Christmas.

D. Online retailers offer better post-Christmas sales.

Questions 14 to 17 are based on the following passage. At the end of the passage, you will be given 20 seconds to answer the questions. Now, listen to the passage.

14. Ballroom dancing used to be associated with

A. TV shows. B. old people.

C. celebrities. D. professional dancers.

15. According to the passage, recent popularity of ballroom dancing is the result of

A. the participation of celebrities.

B. the designing of colourful costumes.

C. the benefits it brings.

D. a TV programme.

16. Which of the following is NOT mentioned about the TV show?

A. Performers have to be formally dressed on the show.

B. Each professional dancer dances with a celebrity.

C. People on the show perform a different dance every week.

D. The show runs for about four months.

17. According to the passage, the TV show has the greatest impact on

A. old people. B. middle-aged people.

C. kids and young people. D. all of the above.

Questions 18 to 20 are based on the following passage. At the end of the passage, you will be given 15 seconds to answer the questions. Now, listen to the passage.

18. According to New Zealand's rules about naming children, which of the following names is

NOT acceptable?

A. Spiderman. B. Gandalf. C. 2win. D. Arsenal.

19. According to the passage, unusual names come from

A. popular culture. B. parents' invention.

C. sports. D. all of the above.

20. All of the following countries have strict rules about naming children EXCEPT

A. Algeria. B. Germany. C. Japan. D. Argentina.

SECTION C NEWS BROADCAST

In this section, you will hear several news items. Listen to them carefully and then answer the questions that follow

Questions 21 to 23 are based on the following news. At the end of the news item, you will be given 15 seconds to answer the questions. Now. listen to the news.

21. Why were the fishing crew stranded on Oct. 10th?

A. They went to a remote area.

B. Their fishing boats collided.

C. They tried to repair their boats.

D. They decided to stay in the boats.

22. How did they survive during those three months?

A. On supplies they brought with them.

B. On supplies sent to them by rescue teams.

C. On supplies left at the military base.

D. Not mentioned in the passage.

23. How were the crew rescued eventually?

A. By helicopter. B. By boat. C. By radio contact. D. By a search team.

Questions 24 and 25 are based on the following news. At the end of the news item, you will be given 10 seconds to answer the questions. Now, listen to the news.

24. Juan Carlos has been King of Spain

A. since 1981. B. for 32 years. C. for 70 years. D. for 17 years.

25. What is the news item mainly about?

A. The King's birthday. B. The stability of the monarchy.

C. Criticism from both the left and the right. D. The King's public defence of his reign.

Questions 26 and27 are based on the following news. At the end of the news item, you will be given 10 seconds to answer the questions. Now, listen to the news.

26. The three suicide bombings occurred in

A. November and December. B. October and November.

C. November. D. December.

27. Did people die in the bombings?

A. No one died in the bombings.

B. Yes. In one of the bombings.

C. Yes. In two of the bombings.

D. Yes. In all the bombings.

Questions 28 to 30 are based on the following news. At the end of the news item, you will be given 15 seconds to answer the questions. Now, listen to the news.

28. What is the purpose of the national survey?

A. To collect data on sources of pollution.

B. To identify pollution in rivers and lakes.

C. To help control environmental pollution.

D. To help control industrial wastes.

29. According to the news item, efforts of environmental protection are especially affected by

A. lack of technology.

B. rapid economic growth.

C. unknown pollution sources.

D. shortage of manpower.

30. Which of the following details is CORRECT according to the news item?

A. Census offices are set up by government departments.

B. A main centre receives reports from provinces.

C. A database is set up for each province.

D. Data will be reviewed and analyzed in mid-2009.

PART III CLOZE [15 MIN]

Decide which of the choices given below would best complete the passage if inserted in the corresponding blanks. Mark the best choice for each blank on Answer Sheet Two.

Scientists around the world are racing to learn how to rapidly diagnose, treat and stop the spread of a new, deadly disease. SARS -- Severe Acute Respiratory Syndrome --- was (31) ____ for the first time in February 2003 in Hanoi, (32) _____ since then has infected more than 1,600 people in 15 countries, killing 63. At this (33) _____, there are more questions than answers surrounding the disease.

Symptoms start (34) _____ a fever over 100.4 degrees F, chills, headache or body (35)____. Within a week, the patient has a dry cough, which might (36) _____ to shortness of breath. In 10% to 20% of cases, patients require (37)_____ ventilation to breathe. About 3.5% die from the disease. Symptoms (38)____ begin in two to seven days, but some reports suggest it (39) ____ take as long as 10 days. Scientists are close to (40) ____ a lab test to diagnose SARS. In the meantime, it is diagnosed by its symptoms. There is no evidence (41)____ antibiotics or anti-viral medicines help, (42) _____ doctors can offer only supportive care. Patients with SARS are kept in isolation to reduce the risk of(43) ____ Scientists aren't sure yet, but some researchers think it's a (44) ____ discovered coronavirus, the family of viruses that cause some common colds.

Most cases appear to have been passed (45) ____ droplets expelled when infected patients cough or sneeze. Family members of infected people and medical workers who care (46) ____ them have been most likely to (47) ____ the illness. But recent developments in Hong Kong suggest that the (48) ____ might spread through air, or that the virus might (49) ____ for two to three hours on doorknobs or other (50)____. Health experts say it is unlikely, though, that sharing an elevator briefly with an infected person would be enough to pass the virus.

(31) A. detected B. caught C. disclosed D. revealed

(32) A. but B. and C. or D. yet

(33) A. time B. point C. aspect D. instance

(34) A. from B. over C. upon D. with

(35) A. hurt B. sore C. aches D. feelings

(36) A. process B. advance C. progress D. convert

(37) A. automatic B. artificial C. mechanical D. controlled

(38) A. regularly B. ordinarily C. traditionally D. generally

(39) A. will B. might C. should D. must

(40) A. cultivating B. fostering C. developing D. designing

(41) A. which B. that C. whether D. what

(42) A. so B. but C. still D. yet

(43) A. communication B. transportation C. transformation D. transmission

(44) A. lately B. newborn C. newly D. renewed

(45) A. under B. through C. beneath D. from

(46) A. for B. over C. after D. about

(47) A. acquire B. receive C. obtain D. contract

(48) A. ailment B. ill-health C. disease D. infection

(49) A. continue B. linger C. delay D. persist

(50) A. exteriors B. outside C. surfaces D. coverings

PART IV GRAMMAR & VOCABULARY [15 MIN]

There are thirty sentences in this section. Beneath each sentence there are four words or phrases marked A, B, C and D. Choose one word or phrase that best completes the sentence. Mark your answers on Answer Sheet Two.

51. What a nice day! How about the three of us _____ a walk in the park nearby?

A. to take B. take C. taking D. to be taking

52. If there were no subjunctive mood, English _____ much easier to learn.

A. could have been B. would'be

C. will be D. would have been

53. She _____ fifty or so when I first met her at a conference.

A. had been B. must be C. has been D. must have been

54. _____ the boss says, it is unreasonable to ask me to work overtime without pay.

A. Whatever B. Whenever C. Whichever D. However

55. A new laptop costs about _____ of a second-hand one.

A. the price of three times B. three times the price

C. as much as the three times price D. three times more than the price

56. I was very interested in _____ she told me.

A. all that B. all which C. all what D. that

57. We consider ______ he should have left without telling anyone beforehand.

A. strange why B. it strange what C. it strange that D. that strange

58. It is going to be fine tomorrow. _______.

A. So is it. B. So it is. C. So it does. D. So does it.

59. Little _____ about her own safety, though she herself was in great danger.

A. she cared B. she may care C. may she care D. did she care

60. The couple had no sooner got to the station _____ the coach left.

A. when B. as C. until D. than

61. Aren't you tired? I ____ you had done enough for today.

A. should have thought B. must have thought

C. might have thought D. could have thought

62. "It seems that she was there at the conference." The sentence means that

A. she seems to be there at the conference.

B. she seemed to be there at the conference.

C. she seems to have been there at the conference.

D. she seemed to being there at the conference.

63. Which of the following adverbs can NOT be used to complete " _____ everybody came"?

A. Nearly B. Quite C. Practically D. Almost

64. In "How much do you think he earns?" how much is ______ of the sentence.

A. the subject B. the adverbial

C. the object D. the complement

65. "The man preparing the documents is the firm's lawyer" has all the following possible meanings EXCEPT

A. the man who has prepared the documents...

B. the man who has been preparing the documents...

C. the man who is preparing the documents...

D. the man who willprepare the documents...

66. During the TV interview, the singer announced that he was going to _____ his new album soon.

A. release B. renew C. relieve D. rehearse

67. Afterworking for the firm for ten years, he finally _____ the rank of deputy director.

A. achieved B. approached C. attained D. acquired

68. Winter is the _____ season at most hotels in this seaside town, because very few tourists come to stay.

A. slow B. slack C. low D. quiet

69. Come on, Jack, tell me the story. Don't keep me in ______.

A. suspense B. suspending C. suspension D. suspender

70. The football match was _____ because of the heavy rain.

A. called over B. called up C. called out D. called off

71. We had a good time there, and the food was plentiful and _____.

A. conducive B. wholesome C. helpful D. appreciative

72. It was strange that she would _____ such an absurd idea.

A. allow B. stick C. take D. entertain

73. The scientists have made an _____ study of the viruses that cause the disease.

A. exhausted B. exhausting C. exhaustive D. exhaustion

74. Do you own your apartment or are you a ______ ?

A. tenant B. customer C. client D. proprietor

75. Representatives from the companies indicated that they should go on working together in _____.

A. unity B. entity C. partners D. partnership

76. We all know that Mary has had a strict _____.

A. growth B. upbringing C. development D. cultivation

77. The drink was packaged in champagne bottles and was being _____ as the real stuff.

A. passed out B. passed by C. passed over D. passed off

78. Last Sunday she came to visit us out of the blue. The italicized phrase means

A. unexpectedly B. unhappily C. untidily D. unofficially

79. The person he interviewed was _____ his former schoolmate.

A. no other than B. no more than C. none other than D. none the less

80. The young employee has a(n)______ quality - he is totally honest.

A. respectable B. admirable C. decent D. approachable

PART V READING COMPREHENSION [25 MIN]

In this section there are four passages followed by questions or unfinished statements, each with four suggested answers marked A, B, C and D. Choose the one that you think is the best answer. Mark your answers on Answer Sheet Two.

TEXT A

Do you realize that every time you take a step, the bones in your hip are subjected to forces between four and five times your body weight? When you are running, this force is increased further still. What happens if through disease a hip-joint ceases to be able to resist such forces? For many years hip-joints and other body joints have been replaceable either partially or completely. It is after all a simple ball and socket joint; it has certain loads imposed on it; it needs reliability over a defined life; it must contain materials suitable for the working environment. Any engineer will recognize these as characteristic of a typical engineering problem, which doctors and engineers have worked together to solve, in order to bring a fresh lease of life to people who would otherwise be disabled.

This typifies the way in which engineers work to help people and create a better quality of life. The fact that this country has the most efficient agricultural industry in the world is another good example. Mechanical engineers have worked with farmers and biologists to produce fertilizers, machinery and harvesting systems. This team effort has now produced crops uniformly waist high or less so that they are better suited to mechanical harvesting. Similar advances with other crops have released people from hard and boring jobs for more creative work, whilst machines harvest crops more efficiently with less waste. Providing more food for the rapidly increasing population is yet another role for the mechanical engineer.

81. According to the passage, when would most weight be imposed on hip-joints?

A. When one is walking. B. When one is running.

C. When one is standing. D. When one is lying down.

82. Engineers regard the replacement of hip-joints as a(n) ____ Problem.

A. mechanical B. medical C. health D. agricultural

83. According to the passage, how do engineers contribute to increasing efficiency of the

agricultural industry?

A. By working with farmers.

B. By working in teams.

C. By growing crops of the same height.

D. By making agricultural machinery.

84. According to the context, "This team effort'" in Paragraph Two refers to

A. mechanical engineers.

B. doctors and engineers.

C. biologists, doctors and farmers.

D. farmers, biologists and engineers.

TEXT B

Nowadays, a cellphone service is available to everyone, everywhere. Probably thousands of people have already been using it, but I just discovered it, so I'm going to claim it and also name it: Fake Foning.

The technology has been working well for me at the office, but there are infinite applications. Virtually in any public space.

Say you work at a big university with lots of talky faculty members buzzing about. Now, say you need to use the restroom. The trip down the hall will take approximately one hour, because a person can't walk into those talky people without getting pulled aside for a question, a bit of gossip, a new read on a certain line of Paradise Lost.

So, a cellphone. Any cellphone. Just pick it up. Don't dial. Just hold that phone to your face and start talking. Walk confidently down the hall engaged in fake conversation, making sure to tailor both the topic and content to the person standing before you whom you are trying to evade.

For standard colleague avoidance, I suggest fake chatting about fake business:

"Yes, I'm glad you called, because we really need to hammer out the details. What's that? Yes, I read Page 12, but if you look at the bottom of 4, I think you can see the problem begins right there."

Be animated. Be engaged in your fake fone conversation. Make eye contact with the people passing, nod to them, gesture keen interest in talking to them at a later time, point to your phone, shrug and move on.

Shoppers should consider fake foning anytime they spot a talky neighbor in the produce department pinching (用手捏) unripe peaches. Without your phone at your face, you'd be in for a 20-minute speech on how terrible the world is.

One important caution about fake foning. The other day I was fake foning my way past a colleague, and he was actually following me to get my attention. I knew he wanted to ask about a project I had not yet finished. I was trying to buy myself some time, so I continued fake foning with my doctor. "So I don't need the operation? Oh, doctor, that is the best news."

And then: Brrrrrrng! Brrrrrmg! Brrrrrmg! My phone started ringing, right there while it was planted on my face. My colleague looked at me, and I at him, and naturally I gasped. "What is the matter with this thing?" I said, pulling the phone away to look at it, and then putting it back to my ear.

"Hello? Are you still there?"

Oops.

85. Which of the following statements is INCORRECT?

A. Cellphone service is popular among people.

B. Cellphone has much use in office.

C. Fake foning is a new cellphone service.

D. Fake foning is a new discovery.

86. What is fake foning?

A. A strategy to avoid people.

B. A device newly produced.

C. A service provided everywhere.

D. A skill of communication.

87. In the author's opinion, in order to make fake foning look real one has to

A. talk about interesting matters.

B. behave politely to people passing by.

C. hold the phone while walking.

D. appear absorbed in conversation.

88. What does the last example show?

A. One effective way is to fake fone one's doctor.

B. One has to be careful while fake foning.

C. Fake foning may not deceive people.

D. Fake foning is always quite successful.

89. After his phone suddenly began ringing, the author

A. immediately started talking to the caller.

B. immediately started talking to his colleague.

C. put the phone away and stopped talking.

D. continued with his fake conversation.

90. What is the tone of the passage?

A. Critical. B. Humorous. C. Serious. D. Unclear.

TEXT C

It was late in the afternoon, and I was putting the final touch on a piece of writing that I was feeling pretty good about. I wanted to save it, but my cursor had frozen. I tried to shut the computer down, and it seized up altogether. Unsure of what else to do, I yanked (用力猛拉) the battery out.

Unfortunately, Windows had been in the midst of a delicate and crucial undertaking. The next morning, when I turned my computer back on, it informed me that a file had been corrupted and Windows would not load. Then, it offered to repair itself by using the Windows Setup CD.

I opened the special drawer where I keep CDs. But no Windows CD in there. I was forced to call the computer company's Global Support Centre. My call was answered by a woman in some unnamed, far-off land. I find it annoying to make small talk with someone when I don't know what continent they're standing on. Suppose I were to comment on the beautiful weather we've been having when there was a monsoon at the other end of the phone? So I got right to the point.

"My computer is telling me a file is corrupted and it wants to fix itself, but I don't have the Windows Setup CD."

"So you're having a problem with your Windows Setup CD." She has apparently been dozing and, having come to just as the sentence ended, was attempting to cover for her inattention.

It quickly became clear that the woman was not a computer technician. Her job was to serve as a gatekeeper, a human shield for the technicians. Her sole duty, as far as I could tell, was to raise global stress levels.

To make me disappear, the woman gave me the phone number for Windows' creator, Microsoft. This is like giving someone the phone number for, I don't know, North America. Besides, the CD worked; I just didn't have it. No matter how many times I repeated my story, we came back to the same place. She was calm and resolutely polite.

When my voice hit a certain decibel (分贝), I was passed along, like a hot, irritable potato, to a technician.

"You don't have the Windows Setup CD, ma'am, because you don't need it," he explained cheerfully.

"Windows came preinstalled on your computer!"

"But I do need it."

"Yes, but you don't have it." We went on like this for a while. Finally, he offered to walk me through the use of a different CD, one that would erase my entire system. "Of course, you'd lose all your e-mail, your documents, your photos." It was like offering to drop a safe on my head to cure my headache. "You might be able to recover them, but it would be expensive." He sounded delighted. "And it's not covered by the warranty (产品保证书)!" The safe began to seem like a good idea, provided it was full.

I hung up the phone and drove my computer to a small, friendly repair place I'd heard about. A smart, helpful man dug out a Windows CD and told me it wouldn't be a problem. An hour later, he called to let me know it was ready. I thanked him, and we chatted about the weather, which was the same outside my window as it was outside his.

91. Why did the author shut down her computer abruptly?

A. She had saved what she had written.

B. She couldn't move the cursor.

C. The computer refused to work.

D. The computer offered to repair itself.

92. Which of the following is the author's opinion about the woman at the Global Support Centre?

A. She sounded helpful and knowledgeable.

B. She was there to make callers frustrated.

C. She was able to solve her computer problem.

D. She was quick to pass her along to a technician.

93. According to the passage, the solution offered by the technician was

A. effective. B. economical. C. unpractical. D. unacceptable.

94. "It was like offering to drop a safe on my head to cure my headache" in the last but one paragraph means that

A. the technician's proposal would make things even worse.

B. the technician's proposal could eventually solve the problem.

C. files stored on her computer were like a safe.

D. erasing the entire system was like curing a headache.

95. It can be inferred from the passage that the differences between the Global Support Centre and the local repair shop lie in all the following EXCEPT

A. efficiency. B. location. C. setup CDs. D. attitude.TEXT D

Not long ago, a mysterious Christmas card dropped through our mail slot. The envelope was addressed to a man named Raoul, who, I was relatively certain, did not live with us. The envelope wasn't sealed, so I opened it. The inside of the card was blank. Ed, my husband, explained that the card was both from and to the newspaper deliveryman. His name was apparently Raoul, and Raoul wanted a holiday tip. We were meant to put a check inside the card and then drop the envelope in the mail. When your services are rendered at 4 a.m., you can't simply hang around, like a hotel bellboy expecting a tip. You have to be direct.

So I wrote a nice holiday greeting to this man who, in my imagination, fires The New York Times from his bike aimed at our front door, causing more noise with mere newsprint than most people manage with sophisticated black market fireworks.

With a start, I realized that perhaps the reason for the 4 a.m. wake-up noise was not ordinary rudeness but carefully executed spite: I had not tipped Raoul in Christmases past. I honestly hadn't realized I was supposed to. This was the first time he'd used the card tactic. So I got out my checkbook. Somewhere along the line, holiday tipping went from an optional thank-you for a year of services to a Mafia-style protection racket (收取保护费的黑社会组织).

Several days later, I was bringing our garbage bins back from the curb when I noticed an envelope taped to one of the lids. The outside of the envelope said MICKEY. It had to be another tip request, this time from our garbage collector. Unlike Raoul, Mickey hadn't enclosed his own Christmas card from me. In a way, I appreciated the directness. "I know you don't care how merry my Christmas is, and that's fine," the gesture said. "I want $30, or I'll 'forget' to empty your garbage bin some hot summer day."

I put a check in the envelope and taped it back to the bin. The next morning, Ed noticed that the envelope was gone, though the trash hadn't yet been picked up: "Someone stole Mickey's tip!" Ed was quite certain. He made me call the bank and cancel the check.

But Ed had been wrong. Two weeks later, Mickey left a letter from the bank on our steps. The letter informed Mickey that the check, which he had tried to cash, had been cancelled. The following Tuesday morning, when Ed saw a truck outside, he ran out with his wallet. "Are you Mickey?"

The man looked at him with scorn. "Mickey is the garbageman. I am the recycling." Not only had Ed insulted this man by hinting that he was a garbageman, but he had obviously neglected to tip him. Ed ran back inside for more funds. Then he noticed that the driver of the truck had been watching the whole transaction. He peeled off another twenty and looked around, waving bills in the air. "Anyone else?"

Had we consulted the website of the Emily Post Institute, this embarrassing breach of etiquette (礼节) could have been avoided. Under "trash/recycling collectors" in the institute's Holiday Tipping Guidelines, it says: "$10 to $30 each." You may or may not wish to know that your pet groomer, hairdresser, mailman and UPS guy all expect a holiday tip.

96. The newspaper deliveryman put a blank card inside the envelope because

A. he forgot to write a few words on it.

B. he wanted the couple to send it back.

C. he used it to ask for a Christmas tip.

D. he was afraid of asking for a tip in person.

97. From the passage, we learn that the author

A. didn't like Raoul's way of delivering the paper.

B. didn't realize why Raoul delivered the paper that way.

C. didn't know that Raoul came very early in the morning.

D. didn't feel it necessary to meet Raoul when he came.

98. According to the passage, the author felt ____ to give Raoul a holiday tip.

A. excited B. delighted C. embarrassed D. forced

99. Which of the following is CORRECT about Mickey, the garbage collector?

A. He wrote a letter to the couple afterwards.

B. He failed to collect the money from the bank.

C. He wanted the couple to send him a Christmas card.

D. He collected both the cheek and the garbage that day.

100. Ed's encounter with the recycling team shows that

A. Ed was desperate to correct his mistake.

B. Ed only wanted to give money to Raoul.

C. Ed was unwilling to tip the truck driver.

D. Ed no longer wanted to give them money.

PART VI WRITING [45 MINI]

TEST FOR ENGLISH MAJORS (2008)

-GRADE FOUR-

TIME LIMIT: 135 MIN

1. When is Anne available for the meeting?

A. The third week of May.

B. The third week of June.

C. The eleventh of June.

D. The eleventh of May.

2. Their meeting will probably take place in ________.

A. London.

B. Toronto.

C. Mexico City.

D. Chicago.

3. When is Eric calling back?

A. Thursday afternoon.

B. Friday afternoon.

C. Thursday morning.

D. Friday morning.

Questions 4 to 6 are based on the following conversation. At the end of the conversation, you will be given 15 seconds to answer the questions. Now, listen to the conversation.

4. According to the woman, advertisements ________.

A. let us know the best product

B. give us sufficient information

C. fail to convince people

D. give misleading information

5. In the woman's opinion, money spent on advertisements is paid ________.

A. by manufacturers

B. by customers

C. by advertisers

D. by all of them

6. Which of the following statements is INCORRECT?

A. The woman seems to be negative about advertising.

B. The woman appears to know more about advertising.

C. The man is to be present at a debate on advertising.

D. The man has a lot to talk about on advertising.

Questions 7 to 10 are based on the following conversation. At the end of the conversation, you will be given 20 seconds to answer the questions. Now, listen to the conversation.

7. Mr. Brown brought with him only a few things because ________.

A. there wasn't enough space in the cupboard

B. the hospital would provide him with everything

C. he was to stay there for a very short time

D. visitors could bring him other things

8. According to the hospital rules, at which of the following hours can visitors see patients?

A. 2:00 pm

B. 5:00 pm

C. 7:00 pm

D. 6:00 pm

9. Which of the following statements is INCORRECT?

A. Patients have breakfast at 8.

B. Patients have lunch at 12.

C. There are special alcohol lounges.

D. There are special smoking lounges.

10. Which statement best describes Mr. Brown?

A. He knows little about hospital rules.

B. He can keep alcohol in the ward.

C. He knows when to smoke.

D. He is used to hospital life.

SECTION B PASSAGES

In this section, you will hear several passages. Listen to the passages carefully and then answer the questions that follow.

Questions 11 to 13 are based on the following passage. At the end of the passage, you will be given 15 seconds to answer the questions. Now, listen to the passage.

11. Meeting rooms of various sizes are needed for ________.

A. contacts with headquarters

B. relaxation and enjoyment

C. informal talks

D. different purposes

12. Which of the following is NOT mentioned in the passage as part of hotel facilities for guests?

A. Restaurants.

B. Cinemas.

C. Swimming pools.

D. Bars.

13. A hotel for an international conference should have the following EXCEPT

A. convenient transport services

B. competent office secretaries

C. good sports and restaurant facilities

D. suitable and comfortable rooms

Questions 14 to 17 are based on the following passage. At the end of the passage, you will be given 20 seconds to answer the questions. Now, listen to the passage.

14. The museum aims mainly to display ________.

A. the area's technological development

B. the nation's important historical events

C. the area's agricultural and industrial development

D. the nation's agricultural and industrial development

15. The following have been significant in the area's prosperity EXCEPT

A. the motorways

B. the Roman road

C. the canals

D. the railways

16. We know from the passage that some exhibits ________.

A. are borrowed from workshops

B. are specially made for display

C. reflect the local culture and customs

D. try to reproduce the scene at that time

17. The passage probably comes from ________.

A. a conversation on the museum

B. a museum tour guide

C. a museum booklet

D. a museum advertisement

Questions 18 to 20 are based on the following passage. At the end of the passage, you will be given 15 seconds to answer the questions. Now, listen to the passage.

18. According to the speaker, safety in dormitory means that you ________.

A. insure all your expensive things

B. lock doors when going out

C. lock windows at night

D. take all necessary precautions

19. What does the speaker suggest girls do when they are going to be out late?

A. Call their friends.

B. Stay with their friends.

C. Avoid walking in streets.

D. Always take a taxi.

20. What is the speaker's last advice?

A. To take a few self-defense classes.

B. To stick to well-lit streets at night.

C. To avoid walking alone at night.

D. To stay with their friends.

SECTION C NEWS BROADCAST

In this section, you will hear several news items. Listen to them carefully and then answer the questions that follow.

Questions 21 and 22 are based on the following news. At the end of the news item, you will be given 10 seconds to answer the questions. Now listen to the news.

21. What happened during the New Year celebration in Thailand?

A. Terrorists fought with Government troops.

B. Thai troops killed terrorists.

C. There were shootings.

D. There were explosions.

22. What has led to the violent situation in the south of Thailand?

A. The Muslims wanted independence.

B. Thai troops have been sent there.

C. About 2000 people have been killed.

D. There have been more bombings since 2004.

Questions 23 and 24 are based on the following news. At the end of the news item, you will be given 10 seconds to answer the questions. Now, listen to the news.

23. Under the national oil law, the Iraqi government ________.

A. will give more oil revenues to only a few provinces

B. will let provinces distribute their oil revenues

C. will distribute oil revenues according to population size

D. will distribute oil revenues according to security needs

24. The construction package is meant to ________.

A. help build more houses

B. help improve the country's economy

C. help more children to go to school

D. help more young people to get education

Questions 25 and 26 are based on the following news. At the end of the news item, you will be given 10 seconds to answer the questions. Now, listen to the news.

25. The joint committee will promote co-operation between Egypt and Spain in all the following areas EXCEPT

A. education

B. industry

C. investment

D. technology

26. What is this news item mainly about?

A. The establishment of a joint committee.

B. The trade relations between Egypt and Spain.

C. The future trade volume between Egypt and Spain.

D. The establishment of a joint business council.

Question 27 is based on the following news. At the end of the news item, you will be given 5 seconds to answer the question. Now, listen to the news.

27. According to the news, Japanese teenage women ________.

A. are less violent than men

B. are less violent than before

C. are more violent than before

D. are more violent than men

Question 28 is based on the following news. At the end of the news item, you will be given 5 seconds to answer the question. Now, listen to the news.

28. Which of the following statements is CORRECT according to the news?

A. Zimbabweans stayed away from voting.

B. Zimbabweans were enthusiastic about voting.

C. Only a few Zimbabweans turned up to vote.

D. Zimbabweans believed that Mugabe would win.

Questions 29 and 30 are based on the following news. At the end of the news item, you will be given 10 seconds to answer the questions. Now, listen to the news.

29. The news mentioned ________ reason (s) for the action taken by Indian telecom workers.

A. 1

B. 2

C. 3

D. 4

30. According to the news, who among the following were NOT affected by telecom workers' action?

A. Banks.

B. Big companies.

C. Long-distance callers.

D. Government officials.

PART III CLOZE (15 MIN)

Decide which of the choices given below would best complete the passage if inserted in the corresponding blanks. Mark the best choice for each blank on your answer sheet.

Salt, shells or metals are still used as money in out-of-the-way parts of the world today.

Salt may seem rather a strange to use as money, in countries where the food of the people is mainly vegetable, it is often an necessity. Cakes of salt, stamped to show their, were used as money in some countries until recent, and cakes of salt buy goods in Borneo and parts of Africa.

Sea shells as money at some time another over the greater part of the Old World. These were mainly from the beaches of the Maldives Islands in the Indian Ocean, and were traded to India and China. In Africa, shells were traded right across the from East to West.

Metal, valued by weight, coins in many parts of the world. Iron, in lumps, bars or rings, is still used in many countries paper money. It can either be exchanged goods, or made into tools, weapons, or ornaments. The early money of China, apart from shells, was of bronze, in flat, round pieces with a hole in the middle, called "cash". The of these are between three thousand and four thousand years old-older than the earliest coins of the eastern Mediterranean.

Nowadays, coins and notes have nearly all the more picturesque of money, and in one or two of the more remote countries people still keep it for future use on ceremonial such as weddings and funerals, examples of money will soon be found only in museums.

31. A. object B. article C. substance D. category

32. A. but B. and C. so D. even

33. A. abstract B. advantageous C. abundant D. absolute

34. A. weight B. value C. role D. size

35. A. times B. events C. situations D. conditions

36. A. ever B. also C. still D. never

37. A. had been used B. are used C. would be used D. would have been used

38. A. and B. but C. yet D. Or

39. A. collected B. produced C. grown D. raised

40. A. city B. district C. communib D. continent

41. A. processed B. produced C. preceded D. proceeded

42. A. in spite of B. instead of C. along with D. in line with

43. A. against B. as C. in D. for

44. A. often B. seldom C. really D. much

45. A. earlier B. earliest C. better D. best

46. A. replace B. reproduced C. reflected D. recovered

47. A. sizes B. shapes C. formats D. forms

48. A. while B. although C. because D. if

49. A. events B. gatherings C. occasions D. assemblies

50. A. original B. primitive C. historical D. crude

PART IV GRAMMAR & VOCABULARY (15 MIN)

There are thirty sentences in this section. Beneath each sentence there are four words or phrase marked A, B, C and D. Choose one word or phrase that best completes the sentence. Mark your answers on your answer sheet.

51. Our association, which has consistently pressed for greater employment opportunities for the disabled, will publish ________ proposals in the near future.

A. their

B. our

C. his

D. its

52. Had Judy been more careful on the maths exam, she ________ much better results now.

A. would be getting

B. could have got

C. must get

D. would get

53. Nine is to three ________ three is to one.

A. when

B. that

C. which

D. what

54. Men differ from animals ________ they can think and speak.

A. for which

B. for that

C. in that

D. in which

55. ________ he wanted to go out with his friends at the weekend, he had to stay behind to finish his assignment.

A. Much though

B. Much as

C. As much

D. Though much

56. I enjoyed myself so much ________ I visited my friends in Paris last year.

A. when

B. which

C. that

D. where

57. Which of the following is INCORRECT?

A. All his lectures were boring.

C. Her few friends are all fond of dancing.

B. Half his money was gone.

D. He invited many his friends to the party.

58. When you have finished with that book, don't forget to put it back on my desk, ________?

A. do you

B. don't you

C. will you

D. won't you

59. What does "He wisely refused to spend his money" mean?

A. It was wise of him to refuse to spend his money.

B. He refused to spend his money in a wise manner.

C. He was short of money and didn't want to buy anything.

D. He refused, in a wise manner, to spend his money.

60. They stood chatting together as easily and naturally as ________.

A. it could be

B. could be

C. it was

D. was

61. The following are all correct responses to "Who told the news to the teacher?" EXCEPT

A. Jim did this.

B. Jim did so.

C. Jim did that.

D. Jim did.

62. Quality is ________ counts most.

A. which

B. that

C. what

D. where

63. In his plays Shakespeare ________ his characters live through their language.

A. would make

B. had made

C. made

D. makes

64. The square itself is five hundred yards wide, five times ________ the size of St. Peter's in ________.

Rome.

A. /

B. that of

C. which is

D. of

65. Which of the following sentences expresses "probability"?

A. You must leave immediately.

B. You must be feeling rather tired.

C. You must be here by eight o'clock.

D. You must complete the reading assignment on time.

66. When he first started in university, he really felt at ________ with his major – economics.

A. shore

B. bank

C. ocean

D. sea

67. On the road motorists should be aware of cyclists and be ________ towards them.

A. considerable

B. considering

C. considerate

D. considered

68. Sally was a bit shy, but the teacher found her quite ________ discussing a recent film with others.

A. at home

B. at most

C. at house

D. at bean

69. The company has capitalized ________ the error of judgment made by its business competitor.

A. in

B. over

C. with

D. on

70. Tim has failed three courses this semester, so he will have to ________ them next semester.

A. remake

B. repeat

C. reapply

D. revise

71. Keep this reference book; it may come in ________ one day.

A. handy

B. useful

C. convenient

D. helpful

72. The questions that the speaker raised were well ________ the average adult.

A. past

B. on

C. beyond

D. through

73. Teachers in this school were encouraged to use drama as a (n) ________ of learning.

A. design

B. instrument

C. agency

D. tool

74. First, we need to find out what his scheme is, and then act ________.

A. sensitively

B. imaginatively

C. efficiently

D. accordingly

75. At first Jim was not quite clear what he was going to do after university, but now he seems ________ on becoming a computer programmer.

A. fit

B. set

C. disposed

D. decided

76. When invited to talk about his achievements+ he refused to blow his own ________ and declined to speak at the meeting.

A. trumpet

B. whistle

C. bugle

D. flute

77. In spite of the treatment, the pain in his leg grew in ________.

A. gravity

B. extent

C. intensity

D. amount

78. Bus services between Town Centre and Newton Housing Estate will be ________ until the motorway is repaired.

A. discontinued

B. suspended

C. halted

D. ceased

79. The moon, being much nearer to the Earth than the Sun, is the ________ cause of the tides.

A. principal

B. basic

C. initial

D. elementary

80. Teddy came to my ________ with a cheque of $200 to pay my room rate, after I phoned him that my wallet had been stolen.

A. attendance

B. assistance

C. rescue

D. safety

PART V READING COMPREHENSION (25 MIN)

TEXT A

When the sun is up in Amsterdam, the largest city in the Netherlands sits quietly on the Amstel River. You can rent a bicycle, visit the Van Gogh or Anne Frank museum, or take a water taxi.

But when the sun goes down, the partying begins. In the big clubs and in coffee shops, tourists gather to hang out, talk politics and smoke.

Several areas of the city clearly show the two worlds that rule Amsterdam. And they're all within a short cab ride of each other.

For example, Dam Square attracts daytime sightseers to its festivals, open markets, concerts and other events. Several beautiful and very popular hotels can be found there. And there is the

Royal Palace and the Magna Plaza shopping mall.

But as evening descends on Dam Square so do the party-seekers. Hip pop or funk music begins blaring from Club Paradiso and Club Melkweg. These are two of the most popular clubs in

Europe. So if you come, be ready to dance. The clubs don't shut down until 4 am.

And while you are there, check out the various inexpensive ways to tour the city. Don't worry about getting lost. Although Dutch is the official language, most people in Amsterdam speak English and are happy to help you with directions.

And you'll notice that half the people in the streets are on bicycles. They rent for US$17 to $20 for a whole day.

Amsterdam also has a good canal system. From anywhere between U852 and $9.50, you can use the canal bus or a water taxi to cruise the "Venice of the North".

You can take in the picturesque canal house architecture: The rows of neat, narrow four-story dwellings of brownstone with large windows are well worth seeing. Many of them are several centuries old.

You might also want to jump out of the canal bus at the Museum Quarter and start walking.

Masterpieces by Dutch artists such as Rembrandt, Bruegel, Van Gogh and others are on display at the Van Gogh Museum, Rembrandt House and others.

The city has an appreciation of its historic past. One place to visit is the Anne Frank House in Nine Streets. It was there that the young Jewish girl wrote her famous diary during World War II. Visitors can view Anne's original diary and climb behind the bookcase to the room where she and her family hid from the Nazis for two years.

81. At the beginning of the passage, the author indicates that ________.

A. Amsterdam is generally known as a quiet city.

B. parties go on all day long in Amsterdam,

C. Amsterdam presents two different pictures.

D. Amsterdam attracts many daytime visitors.

82. Which tourist attraction is cited for elaboration in Paragraphs Four and Five?

A. Royal Palace.

B. Dam Square.

C. Club Paradiso.

D. Magna Plaza.

83. According to the passage, the local people have all the following characteristics EXCEPT

A. they are party goers

B. they show hospitality

C. they can speak English

D. they are fond of cycling

84. Which of the following adjectives can best describe Amsterdam as a tourist city?

A. Modern.

B. Delightful.

C. Quiet.

D. Historic.

TEXT B

In an article some Chinese scholars are described as being "tantalized by the mysterious dragon bone hieroglyphics." Tantalized is one of many English words that have their origins in myths and legends of the past (in this case, Greek and Roman ones). The meaning of the verb tantalize is a very particular one: "to promise or show something desirable to a person and then take it away; to tease by arousing hope." Many (but not all) English dictionaries give you a brief indication of a word's origins in brackets before or after the explanation of the meaning. For tantalize the following explanation is given: [> Tantalus]. This means that you should look up the name Tantalus to find out the word's origins, and if you do, you will find out that in Greek mythology, Tantalus was a king who was punished in the lower world with eternal hunger and thirst; he was put up to his chin in water that always moved away when he tried to drink it and with fruit on branches above him placed just a little bit out of his reach. Can you see why his name was changed into a verb meaning "to tease or torment by arousing desire"?

Another example is the word siren, familiar to us as the mechanical device that makes such an alarming sound when police cars, ambulances, or fire engines approach. This word also has its origins in Greek mythology. The traveler Odysseus (Ulysses to the Romans) made his men plug their ears so that they wouldn't hear the dangerous voices of the sirens, creatures who were half bird and half woman and who lured sailors to their deaths on sharp rocks. So the word came to be associated both with a loud sound and with danger!

When someone speaks of a "jovial mood" or a "Herculean effort," he or she is using words with origins in mythology. Look these words up to find their meaning and relationship to myths.

Many common words, such as the names for the days of the week and the months of the year, also come from mythology. Wednesday derives from the ancient Norse king of the gods, Woden, and Thursday was originally Thor's day, in honour of Thor, the god of thunder. As a matter of fact, all the planets, except the one we live on, bear names that come from Roman mythology, including the planet that is farthest away from the sun and for that reason was called after the Roman god of the dead. This god has also given his name to one of the chemical elements.

Several other elements have names that come from mythology, too.

It seems that myths and legends live on in the English language.

85. The purpose of the first sentence in Paragraph One is ________.

A. to describe the work of some Chinese scholars

B. to arouse readers' interest in hieroglyphics

C. to lead readers onto the main theme

D. to link the preceding part to the present one

86. We learn from the passage, all English dictionaries include ________.

A. legends

B. mythology

C. word origins

D. word definitions

87. The example of tantalize is to show ________.

A. how the word came into existence

B. how Tantalus was punished in the lower world

C. how all English dictionaries show word origins

D. how the meaning of the word changed over the years

88. According to the passage, which of the following does NOT have origins in myths or legends?

A. Jovial.

B. Wednesday.

C. Earth.

D. March.

89. Which of the following can best serve as the title of the passage?

A. Greek and Roman Mythology in Language.

B. Mythological Origins of English Words.

C. Historical Changes in Word Meanings.

D. Mythology and Common Words.

TEXT C

My heart sank when the man at the immigration counter gestured to the back room. I'm an American born and raised, and this was Miami, where I live, but they weren't quite ready to let me in yet.

"Please wait in here, Ms Abujaber," the immigration officer said. My husband, with his very American last name, accompanied me. He was getting used to this. The same thing had happened recently in Canada when I'd flown to Montreal to speak at a book event. That time they held me for 45 minutes. Today we were returning from a literary festival in Jamaica, and I was startled that I was being sent "in back" once again.

The officer behind the counter called me up and said, "Miss, your name looks like the name of someone who's on our wanted list. We're going to have to check you out with Washington."

"How long will it take?"

"Hard to say... a few minutes," he said. "We'll call you when we're ready for you."

After an hour, Washington still hadn't decided anything about me. "Isn't this computerized?"

I asked at the counter. "Can't you just look me up?"

Just a few more minutes, they assured me.

After an hour and a half, I pulled my cell phone out to call the friends I was supposed to meet that evening. An officer rushed over. "No phones!" he said. "For all we know you could be calling a terrorist cell and giving them information."

"I'm just a university professor," I said. My voice came out in a squeak.

"Of course you are. And we take people like you out of here in leg irons every day."

I put my phone away.

My husband and 1 were getting hungry and tired. Whole families had been brought into the waiting room, and the place was packed with excitable children, exhausted parents, even a flight attendant.

I wanted to scream, to jump on a chair and shout: "I'm an American citizen; a novelist; l probably teach English literature to your children." Or would that all be counted against me?

After two hours in detention, I was approached by one of the officers. "You're free to go," he said. No explanation or apologies. For a moment, neither of us moved, we were still in shock.

Then we leaped to our feet.

"Oh, one more thing." He handed me a tattered photocopy with an address on it. "If you weren't happy with your treatment, you can write to this agency."

"Will they respond?" I asked.

"I don't know – I don't know of anyone who's ever written to them before." Then he added,

"By the way, this will probably keep happening each time you travel internationally."

"What can I do to keep it from happening again?"

He smiled the empty smile we'd seen all day. "Absolutely nothing."

After telling several friends about our ordeal, probably the most frequent advice I've heard in response is to change my name. Twenty years ago, my own graduate school writing professor advised me to write under a pen name so that publishers wouldn't stick me in what he called "the ethnic ghetto" – a separate, secondary shelf in the bookstore. But a name is an integral part of anyone's personal and professional identity-just like the town you're born in and the place where you're raised.

Like my father, I'll keep the name, but my airport experience has given me a whole new perspective on what diversity and tolerance are supposed to mean. I had no idea that being an

American would ever be this hard.

90. The author was held at the airport because ________.

A. she and her husband returned from Jamaica

B. her name was similar to a terrorist's

C. she had been held in Montreal

D. she had spoken at a book event

91. She was not allowed to call her friends because ________.

A. her identity hadn't been confirmed yet

B. she had been held for only one hour and a half

C. there were other families in the waiting room

D. she couldn't use her own cell phone

92. We learn from the passage that the author would ________ to prevent similar experience from happening again.

A. write to the agency

B. change her name

C. avoid traveling abroad

D. do nothing

93. Her experiences indicate that there still exists ________ in the US.

A. hatred

B. discrimination

C. tolerance

D. diversity

94. The author sounds in the last paragraph.

A. impatient

B. bitter

C. worried

D. ironic

TEXT D

Public speaking fills most people with dread. Humiliation is the greatest fear; self-exposure and failing to appeal to the audience come a close second. Women hate it most, since girls are pressurized from an early age to be concerned with appearances of all kinds.

Most people have plenty of insecurities, and this seems like a situation that will bring them out. If you were under pressure to be perfect, you are terrified of falling in the most public of ways.

While extroverts will feel less fear before the ordeal, it does not mean they will necessarily do it better. Some very shy people manage to shine. When I met the British comedian Julian Clary, he was shy and cautious, yet his TV performances are perfect.

In fact, personality is not the best predictor of who does it well. Regardless of what you are like in real life, the key seems to be to act yourself.

Actual acting, as in performing the scripted lines of a character other than yourself, does not do the job. While politicians may limit damage by having carefully rehearsed, written scripts to speak from, there is always a hidden awareness among the audience that the words might not be true.

Likewise, the incredibly perfect speeches of many American academics are far from natural.

You may end up buying their book on the way out, but soon afterwards, it is much like fast food, and you get a nameless sense that you've been cheated.

Although, as Earl Spencer proved at his sister Princess Diana's funeral, it is possible both to prepare every word and to act naturally. A script rarely works and it is used to help most speakers.

But, being yourself doesn't work either. If you spoke as if you were in your own kitchen, it would be too authentic, too unaware of the need to communicate with an audience.

I remember going to see British psychiatrist R.

D. Laing speak in public. He behaved like a seriously odd person, talking off the top of his head. Although he was talking about madness and he wrote on mental illness, he seemed to be exhibiting rather than explaining it.

The best psychological place from which to speak is an unselfconscious self-consciousness, providing the illusion of being natural. Studies suggest that this state of "flow", as psychologists call it, is very satisfying.

95. Women hate public speaking most mainly because of ________.

A. their upbringing very early on

B. their inability to appeal to the audience

C. their sense of greater public pressure

D. their sense of greater humiliation

96. "this" in Paragraph Two refers to ________.

A. insecurity

B. sense of failure

C. public speaking

D. pressure

97. Which of the following is NOT the author's viewpoint?

A. Acting like performers spoils the message in a speech.

B. Perfection of scripts is necessary in making good impressions.

C. Acting naturally means less dependence on the prepared script.

D. There should be a balance between actual acting and acting naturally.

98. What is the author's view on personality?

A. Personality is the key to success in public speaking.

B. Extroverts are better public speakers.

C. Introverts have to learn harder to be good speakers.

D. Factors other than personality ensure better performance.

99. The author implies that while speaking R.

D. Laing.

A. was both too casual and authentic

B. was acting like a performer

C. was keeping a good balance

D. was aware of his audience

100. In the last paragraph the author recommends that ________.

A. you forget about your nervousness

B. you feel natural and speak naturally

C. you may feel nervous, but appear naturally

D. you may imagine yourself to be natural

PART VI WRITING (45 MIN)

SECTION A COMPOSITION (35 MIN)

December 5th is International Volunteers Day. Since 1985, when the United Nations announced the special day, tens of millions of people around the world have volunteered to help those in need.

China now has 4.5 million registered volunteers who have provided more than 4.5 billion hours of volunteer work. What can you gain from volunteering?

Write on ANSWER SHEET TWO a composition of about 200 words on the following topic:

The Benefits of Volunteering

You are to write in three parts.

In the first part, state specifically what your opinion is.

In the second part, support your opinion with appropriate details.

In the last part, bring what you have written to a natural conclusion or a summary.

Marks will be awarded for content, organization, grammar and appropriateness. Failure to follow the instructions may result in a loss of marks.

TEST FOR ENGLISH MAJORS (2007)

-GRADE FOUR-

1. Which of the following is NOT needed for the Lost Property Form?

A. Name

B. Nationality

C. Address

D. Phone number

2. From the conversation we know that Mark Adams comes from

A. Essex

B. Edinburgh

C. London

D. The US

3. What will Mark Adams do the day after tomorrow?

A. To come to the office again

B. To wait for the phone call

C. To call the office

D. To write to the office

Questions 4 to 7 are based on the following conversation. At the end of the conversation, you will be given 20 seconds to answer the questions. Now listen to the conversation.

8. At the university Mr. Robinson specialized in

A. maths

B. physics

C. water management

D. geography

9. Mr. Robinson worked for the Indian Government because of

A. university links

B. government agreements

C. company projects.

D. degree requirements

10. After Mr. Robinson returned from India, he

A. changed jobs several times

B. went to live in Manchester

C. did similar work in India

D. became head of a research team

SECTION B PASSAGES

In this section, you will hear several passages. Listen to the passages carefully and then answer the questions that follow. Questions 11-13 are based on the following talk. At the end of the talk, you will be given 15 seconds to answer the questions. Now listen to the talk.

11. According to the talk, the owner of a bike has to

A. register his bike immediately

B. put his bike on a list at once

C. have it stamped with a number

D. report to the police station

12. Thespeaker in the talk recommends

A. two locks for an expensive bike

B. a good lock for an expensive bike

C. cheap locks for cheap bikes

D. good locks for cheap bikes

13. What is the main idea of the talk?

A. How to have the bike stamped

B. How to protect your bike

C. How to buy good locks

D. How to report your lost bike to the police

Questions 14 to 17 are based on the following conversation. At the end of the conversation, you will be given 20 seconds to answer the questions. Now listen to the conversation.

14. Which course(s) runs or run for one hour each time?

A. conversation class

B. writing skills class

C. examination skills class

D. all of the three course

15. which course(s) does or do NOT require enrolment beforehand?

A. conversation class

B. writing skills class

C. examinationa skills class

D. all of the three courses

16. Which course(s) is(are) designed especially for students of economics and social sciences?

A. conversation class

B. writing skills class

C. examination skills class

D. all of the three courses

17. Which course(s) is(are) the shortest?

A. conversation class

B. writing skills class

C. examination skills class

D. all language courses

Questions 18 to 20 are based on the following conversation. At the end of the conversation, you will be given 15 seconds to answer the questions. Now listen to the conversation.

18. How old was Leonardo da Vinci when he moved to Milan?

A. 25

B. 30

C. 35

D. 40

19. Throughout his life, Leonardo da Vinci worked as all the following EXCEPT

A. a painter

B. an engineer

C. an architect

D. a builder

20. Where did Leonard da Vinci die?

A. in France

B. in Milan

C. In Florence

D. in Tuscany

SECTION C NEWS BROADCAST

In this section you will hear several new items. Listen to them carefully and then answer the questions that follow.

Questions 21 to 22 are based on the following news. At the end of the news item, you will be given 10 seconds to answer the questions. Now listen to the news

21. Who had to leave the Caza Strip and the West Bank?

A. The Israeli army

B. The Jewish settlers

C. The Palestinians

D. The Israeli Prime Minister.

22. How many settlements would have to be removed altogether in the Gaza Strip and the West Bank?

A. 2

B. 4

C. 21

D. 25

Questions 23 to 24 are based on the following news. At the end of the news item, you will be given 10 seconds to answer the question. Now listen to the news.

23. Which of the following is NOT mentioned in the news?

A. The agreement has to be approved by Romania

B. The agreement has to be approved by Bulgaria

C. The agreement has to be approved by some EU states

D. The agreement has to be approved by all the EU states

24. Romania and Bulgaria cannot join the EU in 2007 unless they carry out reforms in the following areas EXCEPT

A. manufacturing

B. border control

C. administration

D. justice

Questions 25 to 26 are based on the following news. At the end of the news item, you will be given 10 seconds to answer the question. Now listen to the news.

25. What is the theme of the forum?

A. business leadership

B. global business community

C. economic prospects in China

D. business and government in China

26. According to the news, the first forum was held

A. 10 years ago

B. 3 years ago

C. in 1999

D. in 2001

Questions 27 to 28 are based on the following news. At the end of the news item, you will be given 10 seconds to answer the question. Now listen to the news.

27. About ___ of the 15,000 visitors on the opening day of Hong Kong Disneyland came from the mainland.

A. 4000

B. 5000

C. 6000

D. 7000

28. According to the news, residents in ___showed least interest in visiting the theme park.

A. Beijing

B. Guangzhou

C. Shanghai

D. Hong Kong

Questions 29 to 30 are based on the following news. At the end of the news item, you will be given 10 seconds to answer the question. Now listen to the news.

29. What is the news mainly about?

A. religious violence

B. refugee issues

C. a ferry disaster

D. a rescue operation

30. The ferry boat was designed to carry___passengers.

A. 198

B. 200

C. 290

D. 500

PART III CLOZE (15 MIN)

Decide which of the choices given below would best complete the passage if inserted in the corresponding blanks. Mark the best choice for each blank on your answer sheet.

Until I took Dr Offutt's class in DeMatha High school, I was an underachieving student, but I left that class never to underachieve again. He not only taught me to think, he convinced me, by example as words that it was my moral to do so and to serve others.

of us could know how our relationship would over the years. When I came back to DeMatha to teach English, I worked for Dr Offutt, the department chair. My discussion with him were like graduate seminars in adolescent, classroom management and school leadership.

After several years, I was department chair, and our relationship again. I thought that it might be chairing the department, since all of my English teachers were there, but Dr Offutt supported me. He knew when to give me advice curriculum, texts and personnel, and when to let me my own course.

In 1997, I needed his about leaving DeMatha to become principal at another school. he had asked me to stay at DeMatha, I might have., he encouraged me to seize the opportunity. Five years ago, I became the principal of DeMatha., Dr Offutt was there for me, letting me know that I could him. I have learned from him that great teachers have an inexhaustible of lessons to teach.

31. A. concerned

B. worried

C. determined

D. decided

32. A. as much

B. much as

C. as such

D. such as

33. A. work

B. job

C. duty

D. obligation

34. A. Both

B. Neither.

C. Either.

D. Each.

35. A. evolve

B. stay

C. remain

D. turn

36. A. process

B. procedure

C. development

D. movement

37. A. called

B. named

C. asked

D. invited

38. A. moved

B. altered

C. went

D. shifted

39. A. awkward

B. uneasy

C. unnatural

D. former

40. A. older

B. experienced

C. former

D. /

41. A. /

B. still

C. even

D. already

42. A. through

B. throughout

C. at the beginning

D. all the way

43. A. for

B. at

C. over

D. about

44. A. chart

B. head

C. describe

D. manage

45. A. opinion

B. request

C. permission

D. order

46. A. Even if

B. Although.

C. If

D. When

47. A. Naturally

B. Instead.

C. consequently

D. Still

48. A. Once again

B. Repeatedly.

C. Unusually.

D. Unexpectedly.

49. A. count in

B. count down

C. count out

D. count on

50. A. stock

B. bank

C. wealth

D. store

PART IV GRAMMER &VOCABULARY [15MIN]

There are thirty sentences in this section. Beneath each sentence there are four words or phrases marked A, B, C and D. Choose one word or phrase that best completes the sentences.

51. There are as good fish in the sea ________ ever came out of it.

A. than

B. like

C. as

D. so

52. All the President's Men ________ one of the important books for historians who study the Watergate Scandal.

A. remain

B. remains

C. remained

D. is remaining

53. You ________ borrow my notes provided you take care of them,' I told my friend.

A. could

B. should

C. must

D. can

54. If only the patient ________ a different treatment instead of using the antibiotics, he might still be alive now.

A. had received

B. received

C. should receive

D. were receiving

55. Linda was ________ to experiment a month ago, but she changed her mind at the last minute.

A. to start

B. to have started

C. to be starting

D. to have been starting

56. She ________ fifty or so when I first met her at the conference.

A. must be

B. had been

C. could be

D. must have been

57. It is not ________ much the language as the background that makes the book difficult to understand.

A. that

B. as

C. so

D. very

58. The committee has anticipated the problems that ________ in the road construction project.

A. arise

B. will arise

C. arose

D. have arisen

59. The student said there were a few points in the essay he ________ impossible to comprehend.

A. had found

B. finds

C. has found

D. would find

60. He would have finished his college education, but he ________ to quit and find a job to support his family.

A. had had

B. has

C. had

D. would have

61. The research requires more money than ________.

A. have been put in

B. has been put in

C. being put in

D. to be put in

62. Overpopulation poses a terrible threat to the human race. Yet it is probably ________ a threat to the human race than enviromental destruction.

A. no more

B. not more

C. even more

D. much more

63. It is not uncommon for there ________ problems of communication between the old and the young.

A. being

B. would be

C. be

D. to be

64.________ at in his way, the situation does not seem so desperate.

A. Looking.

B. looked

C. Being looked.

D. to look

65. It is absolutely essential that William ________ his study in spite of some learning difficulties.

A. will continue

B. continued

C. continue

D. continues

66. The painting he bought at the street market the other day was a ________ forgery.

A. man-made

B. natural

C. crude

D. real

67. She's always been kind to me –I can't just turn ________ on her now that she needs my help.

A. my back

B. my head

C. my eye

D. shoulder

68. The bar in the club is for the ________ use of its members.

A. extensive

B. exclusive

C. inclusive

D. comprehensive

69. The tutition fees are ________ to students coming from low-income families.

A. approachable

B. payable

C. reachable

D. affordable

70. The medical experts warned the authorities of the danger of diseases in the ________ of the earthquake.

A. consequence

B. aftermath

C. results

D. effect

71. This sort of rude behaviour in public hardly ________ a person in your position.

A. becomes

B. fits

C. supports

D. improves

72. I must leave now.________, if you want that book I'll bring it next time.

A. Accidentally.

B. Incidentally.

C. Eventually.

D. Naturally.

73. After a long delay, she ________ replying to my e-mail.

A. got away with

B. got back at

C. got back

D. got round to

74. Personal computers are no longer something beyond the ordinary people; they are ________ available these days.

A. promptly

B. instantly

C. readily

D. quickly

75. In my first year at the university I learnt the ________ of journalism.

A. basics

B. basic

C. elementary

D. elements

76. According to the new tax law, any money earned over that level is taxed at the ________ of 59 percent ________.

A. ratio

B. percentage

C. proportion

D. rate

77. Thousands of ________ at the stadium came to their feet to pay tribute to an outstanding performance.

A. audience

B. participants

C. spectators

D. observers

78. We stood still, gazing out over the limitless ________ of the dessert.

A. space

B. expanse

C. stretch

D. land

79. Doctor often ________ uneasiness in the people they deal with.

A. smell

B. hear

C. sense

D. tough

80. Mary sat at the table, looked at the plate and ________ her lips.

A. smacked

B. opened

C. parted

D. seperated

TEXT A

If you like the idea of staying with with a family, living in house might be the answer. Good landladies – those who are superb cooks and launderers, are figures as popular in fiction as the bad ones who terrorize their guest and overcharge them at the slightest opportunity. The truth is probably somewhere between the two extremes. If you are lucky, the food will be adequate, some of your laundry may be done for you and you will have a reasonable amount of comfort and chompanionship. For the less fortune, house rules may restrict the freedom to invite friends to vistit, and shared cooking and bathroom facilities can be frustrating and row-provoking if tidy and untidy guest are living under the same roof.

The same disadvantages can apply to flat sharing, with the added difficulties that arise from deciding who pays for what, and in what proportion. One person may spend hours on the phone, while another rarely makes calls. If you want privacy with guest, how do you persuade the others to go out; how do you persuade them to leave you in peace, especially if you are student and want to study?

Conversely, flat sharing can be very cheap, there will always be someone to talk to and go out with, and the chores, in theory, can be shared.

81. According to the passage, landladies are ________.

A. usually strict

B. always mean

C. adequately competent

D. very popular with their guest

82. What is the additional disadvantage of flat sharing?

A. Problems of sharing and paying.

B. Differences in living habits.

C. Shared cooking and bathroom facilities.

D. Restriction to invite friends to visit.

83. What is NOT mentioned as a benefit of flat sharing?

A. Rent is affordable.

B. There is companionship.

C. Housework.

D. There is peace and quiet.

TEXT B

1. Travelling through the country a couple of weeks ago on business, I was listening to the talk of the late UK writer Douglas Adams' master work "The Hitchhiker's Guide to the Galaxy" on the radio and thought-I know, I'll pick up the next hitchhikers I see and ask them wahat the state of real hitching is today in Britain.

2. I drove and drove on main roads and side roads for the next few days and never saw a single one.

3. When I was in my teens and 20s, hitchhiking was a main form of long-distance transport. The kindness or curiosity of strangers took me all over Europe, North America, Asiaand southern Africa, Some of the lift-givers became friends, many provided hospitality on the road.

4. Not only did you find out much more about a country than when traveling by train or plane, but there was that lelement of excitement about where you would finish up that night.

Hitchhiking featured importantly in Western culture. It has books and songs about it. So what has happened to it?

5. A few years ago, I was asked the same question about hitching in a column of a newspaper. Hundreds of people from all over the world responded with their view on the state of hitchhiking.

6. Rural Ireland was recommended as a friendly place for hitching, as was Quebec, Canada-"if you don't mind being criticized for not speaking French".

7. But while hitchhiking was clearly still alive and well in some places, the general feeling was that throughout much of the west it was doomed.

8. With so much news about crime in the media, people assumed that anyone on the open road without the money for even a bus ticket must present a danger. But do we need to be so wary both to hitch and to give a lift?

9. In Poland in the 1960s, according to a Polish woman who e-mail me,the authorities introduced the Hitchhiker's Booklet. The booklet contained coupons for drivers, so each time a driver picked somebody, he or she received a coupon. At the end of the season, drivers who had picked up the most hikers were rewarded with various prizes. Everyone was hitchhiking then".

10. Surely this is a good idea for society. Hitchhiking would increase respect by breaking down barriers between strangers. It would help fight global warming by cutting down on fuel consumption as hitchhikers would be using existing fuels. It would also improveeducational standards by delivering instant lessons in geography, history, politics and sociology.

11. A century before Douglas Adams wrote his "Hitchhiker's Guide", another adventure story writer, Robert Louis Stevenson, gave us that what should be the hitchhiker's motto:To travel hopefully is a better thing than to arrive." What better time than putting a holiday weekend into practice. Either put it to the test yourself, or help out someone who is trying to travel hopefully with thumb outstreched.

84. In which paragraph (s) does the writer comment on his experience of hitchhiking?

A.

B.

C. and

D. and

85. What is the current situation of hitchhiking?

A. It is popular in some parts of the world.

B. It is popular throughout the west.

C. It is popular in Poland.

86. What is the writer's attitude towards the practice in Poland?

A. Critical.

B. Unclear.

C. Somewhat favourable.

D. Strongly favourable.

87. The writer has mentioned all the following benefits of hitchhiking EXCEPT

A. promoting mutual respect between strangers

B. increasing one's confidence in strangers

C. protecting enviroment

D. enrich one's knowledge

88.Either put it to the test yourself…"in Paragraph means

A. to experience the hopefulness

B. to read Adams' book

C. to offer someone a lift

TEXT C

I am afraid to sleep. I have been afraid to sleep for the last few weeks. I am so tired that, finally, I do sleep, but only for a few minutes. It is not a bad dream that wakes me; it is the realiry I took with me into sleep. I try to think of something else.

Immediately the woman in the marketplace comes into my mind.

I was on my way to dinner last night when I saw her. She was selling skirts. She moved with the same ease and loveliness I often saw in the women of Laos. Her long black hair was as shiny as the black silk of the skirts she was selling. In her hair, she wore three silk ribbons, blue, green, and white. They meminded me of my childhood and how my girlfriends and I used to spend hours braiding ribbons into our hair.

I don't know the word for "ribbons", so I put my hand to my own hair and, with three fingers against my head, I looked at her ribbons and said "Beautiful." She lowered her eyes and said nothing. I wasn't sure if she understood me (I don't speak Laotian very well).

I looked back down at the skirts. They ahd designs in them: squares and triangles and circles of pink and green silk. They were very pretty. I decided to buy one of those skirts, and I began to bargain with her over the price. It is the custom to bargain in Asia. In Laos bargaining is done in soft voices and easy moves with the sort of quiet peacefulness.

She smiled, more with her eyes than with her lips. She was pleased by the few words I was able to say in her language, although they were mostly numbers, and she saw that I understood something about the soft playfulness of bargaining. We shook our heads in disagreement over the price; then, immediately, we made another offer and then another shake of the head. She was so pleased that unexpectedly, she accepted the last offer I made. But it was too soon. The price was too low. She was being too generous and wouldn't make enough money. I moved quickly and picked up two more skirts and paid for all three at the price set; that way I was able to pay her three times as much before she had a chance to lower the price for the larger purchase. She smiled openly then, and, for the first time in months, my spirit lifted. I almost felt happy.

The feeling stayed with me while she wrapped the skirts in a newspaper and handed them to me. When I left, though, the feeling left, too. It was as though it stayed behind in marketplace. I left tears in my throat. I wanted to cry. I didn't, of course.

I have learned to defend myself against what is hard; without knowing it, I have also learned to defend myself against what is soft and what should be easy.

I get up, light a candle and want to look at the skirts. They are still in the newspaper that the woman wrapped them in. I remove the paper, and raise the skirts up to look at them again before I pack them. Something falls to floor. I reach down and feel something cool in my hand. I move close to the candlelight to see what I have. There are five long silk ribbons in my hand, all different colours. The woman in the maketplace! She has given these ribbons to me!

There is no defense against a generous spirit, and this time I cry, and very hard, as if I could make up for all the months that I didn't cry.

90. Which of the following in NOT corret?

A. The writer was not used to bargaining.

B. People in Asia always bargain when buying things.

C. Bargaining in Laos was quiet and peaceful.

D. The writer was ready to bargain with the woman.

91. The writer assumed that the voman accepted the last offer mainly because woman ________.

A. thought that the last offer was reasonable

B. thought she could still make much money

C. was glad that the writer knew their way of bargainning

D. was tired of bargainning with the writer any more

92. Why did the writer finally decide to buy three skirts?

A. The skirts were cheap and pretty.

B. She liked the patterns on the skirts.

C. She wanted to do something as compensation.

D. She was fed up with further bargainning with the woman.

93. When did the writer left the marketplace, she wanted to cry, but did not because ________.

A. she had learned to stay cool and unfeeling

B. she was afraid of crying in public

C. she had learned to face difficulties bravely

D. she had to show in public that she was strong

94. Why did the writer cry eventually when she looked at the skirts again?

A. she suddently felt very sad

B. she liked the ribbons so much

C. she was overcome by emotion

D. she felt sorry for the woman

TEXT D

The kids are hanging out. I pass small bands of students, in my way to work these morings. They have become a familiar part of the summer landscape.

These kids are not old enough for jobs. Nor are they rich enough for camp. They are school children without school. The calendar called the school year ran out on them a few weeks ago. Once supervised by teachers and principals, they now appear to be "self care".

Passing them is like passing through a time zone. For much of our history, after all, Americans arranged the school year around the needs of work and family. In 19th-century cities, schools were open seven or eight hours a day, 11 months a year. In rural America, the year was arranged around the growing season. Now, only 3 percent of families follow the agricultural model, but nearly all schools are scheduled as if our children went home early to milk the cows and took months off to work the crops. Now, three-quarters of the mothers of school-age children work, but the calendar is written as if they were home waiting for the school bus.

The six-hour day, the 180-day school year is regarded as something holy. But when parents work an eight-hour day and a 240-day year, it means something different. It means that many kids go home to empty houses. It means that, in the summer, they hang out.

"We have a huge mismatch between the school calendar and realisties of family life,"says Dr. Ernest Boyer, head of the Carnegie Foundation for the Advancement of Teaching.

Dr. Boyer is one of many who believe that a radical revision of the school calendar is inevitable.School, whether we like it or not, is educational. It always has been."

His is not popular idea. School are routinely burdened with the job of solving all our social problems. Can they be asked to meet the needs of our work and family lives?

It may be easier to promote a linger school year on its educational merits and, indeed, the educational case is compelling. Despite the complaints and studies about our kids' lack of learning, the United State still has a shorter school year than any industrial nation. In most of Europe, the school year is 220 days. In Japan, it is 240 days long. While classroom time alone doesn't produce a well-educated child, learning takes time and more learning takes more time. The long summers of forgetting take a toll.

The opposition to a longer school year comes from families that want to and can provide other experiences for their children. It comes from teachers. It comes from tradition. And surely from kids. But the most important part of the conflict has been over the money.

95. Which of the following is an opinion of the auther's?

A. The kids are hanging out."

B. They are school children without school."

C. These kids are not old enough for jobs."

D. "The calendar called the school year ran out on them a few weeks ago."

96. The current American school calendar was developed in the 19th century according to ________.

A. the growing season on nation's form

B. the labour demands of the industrial age

C. teachers' demands for more vacation time

D. parents' demands for other experiences for their kids

97. The author thinks that the current school calendar ________.

A. is still valid

B. is out of date

C. can not be revised

D. can not be defended

98. Why was Dr. Boy's idea unpopular?

A. He argues for the role of school in solving social problems.

B. He supports the current school calendar.

C. He thinks that school year and family life should be donsidered separately.

D. He strongly believes in the educational role of school.

99."The long summers of forgetting take a toll"in the last paragraph but one means that

A. long summer vacation slows down the progress go learning

B. long summer vacation has been abandoned in Europe

C. long summers result in less learning time

D. long summers are a result of tradition

100. The main purpose of the passage is ________.

A. to describe how American children spend their summer

B. to explain the needs of the modern working families

C. to discuss the problems of the current school calendar

D. to persuade parents to stay at home to look after their kids

TEST FOR ENGLISH MAJORS (2006)

-GRADE FOUR-

TIME LIMIT: 135 MIN

1. How did Mark get there?

A. By train and by car.

B. By plane and by coach.

C. By train and by bus.

D. By bus and by plane.

2. Mark used to wear all the following EXCEPT

A. short hair

B. glasses

C. moustache

D. beard

3. Where is the meeting for new students to be held?

A. In the third room on the right.

B. In the Common Room.

C. In a room at the other end.

D. In Room 501.

Questions 4 to 6 are based on the following conversation. At the end of the conversation, you will be given 15 seconds to answer the questions.

Now, listen to the conversation.

4. What did Steve originally plan to do?

A. To go to a park near the beach.

B. To stay at home.

C. To see a new film.

D. To do some study.

5. Maggie finally decided to go to see a film because ________.

A. there was no park nearby

B. the weather wasn't ideal for a walk

C. it would be easier to go to a cinema

D. Steve hadn't seen the film yet

6. Where did they plan to meet?

A. Outside the Town Hall.

B. Near the bank.

C. In Steve's place.

D. At the cinema.

Questions 7 to 10 are based on the following conversation. At the end of the conversation, you will be given 20 seconds to answer the questions.

Now, listen to the conversation.

7. The following details are true about the new device EXCEPT

A. it has color

B. it has a moving image

C. it costs less money

D. it is not on the market

8. Why didn't Bill want one of them?

A. He wanted to buy one from Japan.

B. He wasn't sure about its quality.

C. He thought it was for business use.

D. He thought it was expensive.

9. Which of the following statements is INCORRECT about the woman?

A. She had never read the magazine herself.

B. She knew who usually read the magazine.

C. She was quite interested in the new device.

D. She agreed with Bill at the end of the conversation.

10. The conversation is mainly about ________.

A. a new type of telephone

B. the cost of telephones

C. some features of the magazine

D. the readership of the magazine

SECTION B PASSAGES

In this section, you will hear several passages. Listen to the passages carefully and then answer the questions that follow.

Questions 11 to 13 are based on the following passage. At the end of the passage, you will be given 15 seconds to answer the questions.

Now, listen to the passage.

11. In the old days dogs were used for the following EXCEPT

A. hunting other animals

B. driving sheep

C. guarding chickens

D. keeping thieves away

12. Which of the following is CORRECT?

A. Dogs are now treated as part of a family.

B. Dogs still perform all the duties they used to do.

C. People now keep dogs for the same reasons as before.

D. Only old people are seen walking their dogs.

13. The passage is mainly about.

A. what dogs can do

B. how to keep dogs

C. dogs and their masters

D. reasons for keeping dogs

Questions 14 to 17 are based on the following passage. At the end of the passage, you will be given 20 seconds to answer the questions.

Now, listen to the passage.

14. According to the passage, the working conditions in the new place ________.

A. are the same as the speaker is used to

B. are expected to be rather poor

C. are just as adequate

D. are not yet clear

15. What is the speaker going to do in the new place?

A. Traveling.

B. Studying.

C. Settling down.

D. Teaching.

16. The speaker expects ________.

A. fewer choices of food

B. many ways to do washing

C. modern lighting facilities

D. new types of drinking water

17. From the passage we can learn that the speaker ________.

A. is unprepared for the new post

B. is unclear about the conditions there

C. is ready for all the difficulties there

D. is eager to know more about the post

Questions 18 to 20 are based on the following passage. At the end of the passage, you will be given 15 seconds to answer the questions.

Now, listen to the passage.

18. According to the passage, when are children first expected to study hard?

A. Before 6 years of age.

B. Between 6 and 10.

C. After l0 years of age.

D. After 12 years of age.

19. Parents who abuse their children tend to have the following problems EXCEPT

A. religious problems

B. emotional problems

C. financial problems

D. marriage problems

20. Which of the following statements is CORRECT?

A. Boys and girls are equally energetic.

B. Parents have higher expectations for boys.

C. Some parents lack skills to deal with their kids.

D. Some parents are ill-educated and ill-tempered.

SECTION CNEWS BROADCAST

In this section, you will hear several news items. Listen to them carefully and then answer the questions that follow.

Questions 21 and 22 are based on the following news. At the end of the news item, you will be given 10 seconds to answer the questions.

Now, listen to the news.

21. What has happened to the Cubans?

A. They set foot in Florida.

B. They were drowned.

C. They were flown to the U.S.

D. They were sent back to Cuba.

22. How did the Cubans try to enter the U.S.?

A. In a small boat.

B. In an old truck.

C. By swimming.

D. By driving.

Question 23 is based on the following news. At the end of the news item, you will be given 5 seconds to answer the question.

Now, listen to the news.

23. How many cities will have air quality monitoring systems installed by the end of this year?

A. 42 cities.

B. 220 cities.

C. 150 cities.

D. 262 cities.

Questions 24 and 25 are based on the following news. At the end of the news item, you will be given 10 seconds to answer the questions.

Now, listen to the news.

24. Altogether how many were reported missing?

A. 68.

B. 90.

C. 150.

D. 40.

25. Which of the following details is INCORRECT?

A. The two ferries sank on different days.

B. The accidents were caused by storms.

C. The two ferries sank on the same river.

D. More people were rescued from the first ferry.

Question 26 is based on the following news. At the end of the news item, you will be given 5 seconds to answer the question.

Now, listen to the news.

26. What is the news item mainly about?

A. Indonesian government policies.

B. Australia's support to the U.N. assistance mission.

C. Opening of an Australian consulate in East Timor.

D. Talks between Australia and Indonesia.

Questions 27 and 28 are based on the following news. At the end of the news item, you will be given 10 seconds to answer the questions.

Now, listen to the news.

27. The news item is mainly about a joint venture between.

A. a U.S. company and a U.K. company

B. a Swiss company and a U.K. company

C. two Taiwanese companies

D. a mainland company and a U.S. company

28. Who will provide the distribution networks in the joint venture?

A. Unilever.

B. Nestle.

C. PepsiCo.

D. Coca Cola.

Questions 29 and 30 are based on the following news. At the end of the news item, you will be given 10 seconds to answer the questions.

Now, listen to the news.

29. Who staged the protest on Saturday?

A. The soldier.

B. The peace camp.

C. The militants.

D. The hardliners.

30. Which of the following details about the news is INCORRECT?

A. 13 soldiers were killed last week.

B. 100,000 people participated in the protest.

C. The protesters demanded a pullout from Gaza.

D. The Prime Minister rejected the pullout plan.

PART III CLOZE

There are many superstitions in Britain, but one of the most held is that it is unlucky to walk under a ladder even if it means the pavement into a busy street! you must pass under a ladder you can bad luck by crossing your fingers and them crossed until you have seen a dog., you may lick your finger and a cross on the toe of your shoe, and not look again at the shoe until the has dried.

Another common is that it is unlucky to open an umbrella in the house-it will either bring to the person who opened it or to the whole. Anyone opening an umbrella in fine weather is, as it inevitably brings rain!

The number 13 is said to be unlucky for some, and when the 13th day of the month on a Friday, anyone wishing to avoid a bad event had better stay. the worst misfortune that can happen to a person is caused by breaking a mirror, it brings seven years of bad luck! The superstition is supposed to in ancient times, when mirrors were considered to be tools of the gods.

Black cats are generally considered lucky in Britain, even though they are witchcraft.. it is lucky if a black cat crosses your path-although in America the exact opposite belief prevails.

Finally, a commonly held superstition is that of touching wood luck. This measure is most often taken if you think you have said something that is tempting fate, such as "my car has never, touch wood?"

31. A broadly B widely C quickly D speedily ________.

32. A running from B jumping off C stepping off D keeping from ________.

33. A If B As C Though D Unless ________.

34. A erase B remove C avoid D ease ________.

35. A keep B keeping C kept D to keep ________.

36. A Consequently B However C Comparatively D Alternatively ________.

37. A make B print C perform D produce ________.

38. A label B symbol C mark D cut ________.

39. A argument B superstition C opinion D idea ________.

40. A loss B difficulty C tragedy D misfortune ________.

41. A house B household C home D circle ________.

42. A unwise B unintelligent C unpopular D unfortunate ________.

43. A falls B arrives C drops D happens ________.

44. A away B outdoors C indoors D far ________.

45. A when B as C if D though ________.

46. A have originated B be originating C be originated D originate ________.

47. A concerned about B related with C associated with D connected in ________.

48. A especially B specially C frequently D rarely ________.

49. A as B for C in D of ________.

50. A broken up B broken off C broken away D broken down ________.

PART IV GRAMMAR and VOCABULARY

51. ________ dull he may be, he is certainly a very successful top executive.

A. Although.

B. whatever

C. As

D. However.

52. If only I ________ play the guitar as well as you!

A. would

B. could

C. should

D. might

53. The party, ________ I was the guest of honour, was extremely enjoyable.

A. by which

B. for which

C. to which

D. at which

54. It's high time we ________ cutting down the rainforests.

A. stopped

B. had to stop

C. shall stop

D. stop

55. The student said there were a few points in the essay he ________ impossible to comprehend.

A. has found

B. was finding

C. had found

D. would find

56. Loudspeakers were fixed in the hall so that everyone ________ an opportunity to hear the speech.

A. ought to have

B. must have

C. may have

D. should have

57. I am surprised ________ this city is a dull place to live in.

A. that you should think

B. by what you are thinking

C. that you would think

D. with what you were thinking

58. Susan is very hardworking, but her pay is not ________ for her work.

A. enough good

B. good enough

C. as good enough

D. good as enough

59. It is imperative that the government ________ more investment into the shipbuilding industry.

A. attracts

B. shall attract

C. attract

D. has to

60. Land belongs to the city; there is ________ thing as private ownership of land.

A. no such a

B. not such

C. not such a

D. no such

61. My daughter has walked eight miles today. We never guessed that she could walk ________ far.

A. /

B. such

C. that

D. as

6 The statistics ________ that living standards in the area have improved drastically in recent times.

A. proves

B. is proving

C. are proving

D. prove

63. There are only ten apples left in the baskets, ________ the spoilt ones.

A. not counting

B. not to count

C. don't count

D. having not counted

64. It was ________ we had hoped ________.

A. more a success than

B. a success more than

C. as much of a success as

D. a success as much as

65. There used to be a petrol station near the park, ________?

A. didn't it

B. doesn't there

C. usedn't it?

D. didn't there

66. It is an offence to show ________ against people of different races.

A. distinction

B. difference

C. separation

D. discrimination

67. A great amount of work has gone into ________ the Cathedral to its previous splendour.

A. refreshing

B. restoring

C. renovating

D. renewing

68. The thieves fled with the local police close on their ________.

A. backs

B. necks

C. toes

D. heels

69. The economic recession has meant that job ________ is a rare thing.

A. security

B. safety

C. protection

D. secureness

70. Many people nowadays save money to ________ for their old age.

A. cater

B. supply

C. provide

D. equip

71. The tone of the article ________ the writer's mood at the time.

A. reproduced

B. reflected

C. imagined

D. imitated

72. This is not the right ________ to ask for my help; I am far too busy even to listen ________.

73. The job of a student accommodation officer ________ a great many visits to landladies.

A. concerns

B. offers

C. asks

D. involves

74. Our family doctor's clinic ________ at the junction of two busy roads.

A. rests

B. stands

C. stays

D. seats

75. She was so fat that she could only just ________ through the door.

A. assemble

B. appear

C. squeeze

D. gather

76. After the heavy rain, a builder was called to repair the roof, which was ________.

A. leaking

B. trickling

C. prominent

D. noticeable

77. The reception was attended by ________ members of the local community.

A. excellent

B. conspicuous

C. prominent

D. noticeable

78. Share prices on the Stock Exchange plunged sharply in the morning but ________ slightly in the afternoon.

A. regained

B. recovered

C. restored

D. revived

79. His brain has worked away on the idea of a universal cure.

A. rich

B. quick

C. productive

D. fertile

80. The couple has donated a not ________ amount of money to the foundation.

A. inconsiderable

B. inconsiderate

C. inaccurate

D. incomparable

PART V READING COMPREHENSION

阅读

TEXT A

In the case of mobile phones, change is everything. Recent research indicates that the mobile phone is changing not only our culture, but our very bodies as well.

First. Let's talk about culture. The difference between the mobile phone and its parent, the fixed-line phone, you get whoever answers it.

This has several implications. The most common one, however, and perhaps the thing that has changed our culture forever, is the "meeting" influence. People no longer need to make firm plans about when and where to meet. Twenty years ago, a Friday night would need to be arranged in advance. You needed enough time to allow everyone to get from their place of work to the first meeting place. Now, however, a night out can be arranged on the run. It is no longer "see you there at 8", but "text me around 8 and we'll see where we all are".

Texting changes people as well. In their paper, "insights into the Social and Psychological Effects of SMS Text Messaging", two British researchers distinguished between two types of mobile phone users: the "talkers" and the "texters"-those who prefer voice to text message and those who prefer text to voice.

They found that the mobile phone's individuality and privacy gave texters the ability to express a whole new outer personality. Texters were likely to report that their family would be surprised if they were to read their texts. This suggests that texting allowed texters to present a self-image that differed from the one familiar to those who knew them well.

Another scientist wrote of the changes that mobiles have brought to body language. There are two kinds that people use while speaking on the phone. There is the "speakeasy": the head is held high, in a self-confident way, chatting away. And there is the "spacemaker": these people focus on themselves and keep out other people.

Who can blame them? Phone meetings get cancelled or reformed and camera-phones intrude on people's privacy. So, it is understandable if your mobile makes you nervous. But perhaps you needn't worry so much. After all, it is good to talk

81. When people plan to meet nowadays, they ________.

A. arrange the meeting place beforehand

B. postpone fixing the place till last minute

C. seldom care about when and where to meet

D. still love to work out detailed meeting plans.

82. According to the two British researchers, the social and psychological effect are mostly likely to be seen on ________.

A. talkers

B; the "speakeasy"

c. the "spacemaker"

D. texters

83. We can infer from the passage that the texts sent by texters are ________.

A. quite revealing

B. well written

C. unacceptable by others

D. shocking to others

84. According to the passage, who is afraid of being heard while talking on the mobile ________.

A. talkers

B. the speakeasy

C. the spacemaker

D. texters

85. An appropriate title for the passage might be ________.

A. The SMS Effect.

B. Cultural Implication of Mobile Use.

C. Change in the Use of the Mobile.

D. Body Language and the Mobile Phone.

TEXT B

Over the last 25 years, British society has changed a great deal – or at least many parts of it have. In some ways, however, very little has changed, particularly where attitudes are concerned. Ideas about social class – whether a person is "working-class" or "middle-class" – are one area in which changes have been extremely slow.

In the past, the working-class tended to be paid less than middle-class people, such as teachers and doctors. As a result of this and also of the fact that workers' jobs were generally much less secure, distinct differences in life-styles and attitudes came into existence. The typical working man would collect his wages on Friday evening and then, it was widely believed, having given his wife her "housekeeping", would go out and squander the rest on beer and betting.

The stereotype of what a middle-class man did with his money was perhaps nearer the truth. He was-and still is – inclined to take a longer-term view. Not only did he regard buying a house of these provided him and his family with security. Only in very few cases did workers have the opportunity (or the education and training) to make such long-term plans.

Nowadays, a great deal has changed. In a large number of cases factory workers earn as much, if not more, than their middle-class supervisors. Social security and laws to improve century, have made it less necessary than before to worry about "tomorrow". Working-class people seem slowly to be losing the feeling of inferiority they had in the past. In fact there has been a growing tendency in the past few years for the middle-classes to feel slightly ashamed of their position.

The changes in both life-styles and attitudes are probably most easily seen amongst younger people. They generally tend to share very similar tastes in music and clothes, they spend their money in having a good time, and save for holidays or longer-term plans when necessary. There seems to be much less difference than in precious generations. Nevertheless, we still have a wide gap between the well-paid (whatever the type of job they may have) and the low-paid. As long as this gap exists, there will always be a possibility that new conflicts and jealousies will emerge, or rather that the old conflicts will re-appear, but between different groups.

86. Which of the following is seen as the cause of class differences in the past?

A. life style and occupation

B. Attitude and income

C. income and job security

D. job security and hobbies

87. The writer seems to suggest that the description of – is closer to truth?

A. middle –class ways of spending money

B. working-class ways of spending the weekend

C. working-class drinking habits

D. middle-class attitudes

88. According to the passage, which of the following is not a typical feature of the middle-class?

A. desiring for security

B. Making long term plans

C. having priorities in life

D. saving money

89. Working-class people's sense of security increased as a result of all the following factor EXCEPT?

A. better social security

B. more job opportunities

C. higher living standard

D. better legal protection.

90. Which of the following statement is incorrect?

A. Changes are slowly taking place in all sectors of the British society.

B. The gap between working-class and middle-class young people is narrowing.

C. Different in income will remain but those in occupation will disappear.

D. Middle-class people may sometimes feel inferior to working-class people.

TEXT C

For several days I saw little of Mr. Rochester. In the morning he seemed much occupied with business, and in the afternoon gentlemen from the neighbourhood called and some times stayed to dine with him. When his foot was well enough, he rode out a great deal.

During this time, all my knowledge of him was limited to occasional meetings about the house, when he would sometimes pass me coldly, and sometimes bow and smile. His changes of manner did not offend me, because I saw that I had nothing to do with the cause of them.

One evening, several days later, I was invited to talk to Mr. Rochester after dinner. He was sitting in his armchair, and looked not quite so severe, and much less gloomy. There was a smile on his lips, and his eyes were bright, probably with wine. As I was looking at him, he suddenly turned, and asked me, "do you think I'm handsome, Miss Eyre?"

The answer somehow slipped from my tongue before I realized it: 'No, sir."

"ah, you really are unusual! You are a quiet, serious little person, but you can be almost rude."

"Sir, I'm sorry. I should have said that beauty doesn't matter, or something like that,"

"no, you shouldn't! I see, you criticize my appearance, and then you stab me in the back! You have honesty and feeling. There are not many girls like you. But perhaps I go too fast. Perhaps you have awaful faults to counterbalance your few good points

I thought to myself that he might have too. He seemed to read my mind, and said quickly," yes, you're right. I have plenty of faults. I went the wrong way when I was twenty-one, and have never found the right path again. I might have been very different. I might have been as good as you, and perhaps wiser. I am not a bad man, take my word for it, but I have done wrong. It wasn't my character, but circumstances which were to blame. Why do I tell you all this? Because you're the sort of person people tell their problems and secrets to, because you're sympathetic and give them hope."

It seemed he had quite a lot to talk to me. He didn't seem to like to finish the talk quickly, as was the case for the first time.

"Don't be afraid of me, Miss Eyre." He continued. "you don't relax or laugh very much, perhaps because of the effect Lowood school has had on you. But in time you will be more natural with me, and laugh, and speak freely. You're like a bird in a cage. When you get out of the cage, you'll fly very high. Good night."

91: at the beginning miss Eyre 's impressions of Mr. Rochester were all except

A. busy

B. sociable

C. friendly

D. changeable

92. in ".... and all my knowledge him was limited to occasional meetings about the house,…". the word about means ________.

A. around

B. on

C. outside

D. concerning.

93. Why did Mr. Rochester say "... and the you stab me in the back!" in the 7th para.?

A. because Jane had intended to kill him with a knife

B. because Jane had intended to be more critical.

C. because Jane had regretted having talked to him

D. because Jane had said something else to correct herself.

94. From what Mr. Rochest told Miss Eyre, we can conclude that he wanted to ________.

A. tell her all his troubles

B. tell her his life experience.

C. change her opinion of him

D. change his circumstances

95. At the end of the passage, Mr. Rochester sounded ________.

A. rude

B. cold

C. friendly

D. encouraging.

TEXT D

The ideal companion machine-the computer-would not only look, feel, and sound friendly but would also be programmed to behave in a pleasant manner. Those qualities that make interaction comfortable, and yet the machine would remain slightly unpredictable and therefore interesting. In its first encounter it might be somewhat hesitant, but as it came to know the user it would progress to a more relaxed and intimate style. The machine would not be a passive participant but would add its own suggestions, information, and opinions; it would sometimes take the initiative in developing or changing the topic and would have a personality of its own.

Friendships are not made in a day, and the computer would be more acceptable as a friend if it imitated the gradual changes that occur when one person is getting to know another. At an appropriate time it might also express the kind of affection that stimulates attachment and intimacy. The whole process would be accomplished in a subtle way to avoid giving an impression of over-familiarity that would be likely to produce irritation. After experiencing a wealth of powerful, well-timed friendship indicators, the user would be very likely to accept the computer as far more than a machine and might well come to regard it as a friend.

An artificial relationship of this type would provide many of the benefits that could continue from previous discussions. It would have a familiarity with the user's life as revealed in earlier contact, and it would be understanding and good-humored. The computer's own personality would be lively and impressive, and it would develop in response to that of the user. With features such as these, the machine might indeed become a very attractive social partner.

96. Which of the following is not a feature of the ideal companion machine?

A. Active in communication

B. Attractive in personality.

C. enjoyable in performance

D. unpredictable in behaviour

97. The computer would develop friendships with humans in a (n) – way.

A. Quick

B. unpredictable

C. productive

D. inconspicuous.

98. Which of the following aspects is not mentioned when the passage discusses the benefits of artificial relationships?

A. Being able to pick up an interesting conversation.

B. Being sensitive to earlier contact.

C. Being ready to learn about the person's life

D. Having a pleasant and adaptable personality.

99. throughout the passage, the author is ________ in his attitude toward the computer ________.

A. favourable

B. critical

C. vague

D. hesitant

100. Which might be the most appropriate title of the passage?

A. Articial relationshios.

B. How to form intimate relationships

C. The affectionate machine

D. Humans and computers

TEST FOR ENGLISH MAJORS (2005)

-GRADE FOUR-

Questions 1 to 3 are based on the following conversation. At the end of the conversation, you will be given 15 seconds to answer the questions. Now listen to the conversation.

1. According to the conversation, Mr. Johnson is NOT very strong in ________.

A. history

B. geography

C. mathematics

D. art

2. Mr. Johnson thinks that ________ can help him a lot in the job.

A. logic

B. writing

C. history

D. mathematics

3. Mr. Johnson would like to work as a (n)

A. adviser

B. computer programmer

C. product designer

D. school teacher

Questions 4 to 7 are based on the following conversation. At the end of the conversation, you will be given 20 seconds to answer the questions. Now listen to the conversation.

4. What is the main purpose of the research?

A. To make preparations for a new publication.

B. To learn how couples spend their weekends.

C. To know how housework is shared.

D. To investigate what people do at the weekend.

5. What does the man do on Fridays?

A. He goes to exercise classes.

B. He goes sailing.

C. He goes to the cinema.

D. He stays at home.

6. On which day does the couple always go out?

A. Friday.

B. Saturday.

C. Sunday.

D. Any weekday.

7. Which personal detail does the man give?

A. Surname.

B. First name.

C. Address.

D. Age.

Questions 8 to 10 are based on the following conversation. At the end of the conversation, you will be given 15 seconds to answer the questions. Now listen to the conversation.

8. Parcel Express needs the following details about the sender EXCEPT

A. name

B. address

C. receipt

D. phone number

9. Parcels must be left open mainly for ________.

A. customs' check

B. security check

C. convenience's sake

D. the company's sake

10. The woman's last inquiry is mainly concerned with ________.

A. the time needed for sending the parcel

B. the flight time to New York

C. the parcel destination

D. parcel collection

SECTION B PASSAGES

In this section, you will hear several passages. Listen to the passages carefully and then answer the questions that follow.

Questions 11 to 13 are based on the following passage. At the end of the passage, you will be given 15 seconds to answer the questions. Now listen to the passage.

11. Where is the train to Nanjing now standing?

A. At Platform 7.

B. At Platform 8.

C. At Platform 9.

D. At Platform 13.

12. Which train will now leave at 11:35?

A. The train to Jinnan.

B. The train to Zhengzhou.

C. The train to Tianjin.

D. The train to Hangzhou.

13. Which train has now been cancelled?

A. The train to Jinnan.

B. The train to Zhengzhou.

C. The train to Tianjin.

D. The train to Hangzhou.

Questions 14 to 17 are based on the following passage. At the end of the passage, you will be given 20 seconds to answer the questions. Now listen to the passage.

14. The museum was built in memory of those ________.

A. who died in wars

B. who worked to help victims

C. who lost their families in disasters

D. who fought in wars

15. Henry Durant put forward the idea because he ________.

A. had once fought in a war in Italy

B. had been wounded in a war

C. had assisted in treating the wounded

D. had seen the casualties and cruelties of war

16. Which of the following statements about the symbols is INCORRECT?

A. Both are used as the organization's official symbols.

B. Both are used regardless of religious significance.

C. The red cross was the organization's original symbol.

D. The red crescent was later adopted for use in certain regions.

17. How should cheerleading be viewed according to the passage?

A. It is just a lot of cheering.

B. It mainly involves yelling.

C. It mainly involves dancing.

D. It is competitive in nature.

Questions 18 to 20 are based on the following passage. At the end of the passage, you will be given 15 seconds to answer the questions. Now listen to the passage.

18. How do the cheerleaders perform their jobs?

A. They set fireworks for their team.

B. They put on athletic shows.

C. They run around the spectators.

D. They yell for people to buy drinks.

19. Why do the cheerleaders sometimes suffer physical injuries?

A. Because they try dangerous acts to catch people's attention.

B. Because they shout and yell so their voice becomes hoarse.

C. Because they go to the pyramid and the hills to perform.

D. Because they dance too much every day for practice.

20. Which of the following statements is NOT true?

A. The first cheerleaders was a man named John Campbell.

B. Cheerleaders' contests are only held at the state level.

C. Before 1930 there were no women cheerleaders.

D. The first cheerleading occurred in 1898.

SECTION C NEWS BROAOCAST

Questions 21 to 22 are based on the following news. At the end of the news item, you will be given 10 seconds to answer the questions. Now listen to the news.

21. How many of the emigrants died after being thrown into the sea?

A. 15 of them

B. 3 of them

C. 100 of them

D. Dozens of them.

22. The illegal emigrants came from ________.

A. Italy.

B. Africa.

C. the Mediterranean region

D. places unknown

Question 23 is based on the following news. At the end of the news item, you will be given 5 seconds to answer the question. Now listen to the news.

23. What does the news item mainly report?

A. China will send three people into space in a week.

B. Three Chinese astronauts will spend a week in space.

C. The Shenzhou VI will be launched next year.

D. Shenzhou V circled the earth for two days.

Questions 24 and 25 are based on the following news. At the end of the news item, you will be given 10 seconds to answer the questions. Now listen to the news.

24. Which of the following had NOT been affected by the wildfires?

A. Houses.

B. Land.

C. Skies.

D. Cars.

25. The fires were thought to have been started ________.

A. purposefully

B. accidentally

C. on the Mexican border

D. in southern California

Questions 26 to 28 are based on the following news. At the end of the news item, you will be given 15 seconds to answer the question. Now listen to the news.

26. ________ ranks second among leading tourism nations.

A. France.

B. The United States.

C. Spain.

D. Italy.

27. It is predicted that by 2020 China will receive ________ visitors.

A. 77 million

B. 130 million

C. 36.8 million

D. 100 million

28. According to a Xinhua report, last year saw a ________ per cent increase in the number of Chinese traveling abroad.

A. 16.6

B. 30

C. 100

D. 37

Question 29 and 30 are based on the following news. At the end of the news item, you will be given 10 seconds to answer the question. Now listen to the news.

29. What would happen to the Argentine officers?

A. They would be arrested by Spanish authorities.

B. They would be tried in an Argentine court.

C. They would be sent to Spain for trial.

D. They would be tortured or murdered.

30. What accusation would the Argentine officers face?

A. Violation of human rights.

B. Involvement in illegal actions.

C. Planning anti-government activities.

D. Being part of the military rule.

PART III CLOZE (15 MIN)

Decide which of the choices given below would best complete the passage if inserted in the corresponding blanks. Mark the best choice for each blank on your answer sheet.

A person's home is as much a reflection of his personality as the clothes he wears, the food he eats and the friends with whom he spends his time. Depending on personality, most have in mind a (n) " home". But in general, and especially for the student or new wage earners, there are practical of cash and location on achieving that idea.

Cash, in fact, often means that the only way of

when you leave school is to stay at home for a while until things

financially. There are obvious of living at home – personal laundry is usually done along with the family wash; meals are provided and there will be a well-established circle of friends to. And there is the responsibility for paying bills, rates, etc.

On the other hand, depends on how a family gets on. Do your parents like your friends? You may love your family –do you like them? Are you prepared to be when your parents ask where you are going in the evening and what time you expect to be back? If you find that you cannot manage a (n), and that you finally have the money to leave, how do you finding somewhere else to live?

If you plan to stay in your home area, the possibilities are

________ well-known to you already. Friends and the local paper are always

________. If you are going to work in a area, again there are the papers – and the accommodation agencies, these should be approached with. Agencies are allowed to charge a fee, usually the of the first week's rent, if you take accommodation they have found for you.

31. A. ideal

B. perfect

C. imaginary

D. satisfactory

32. A. deficiencies

B. weaknesses

C. insufficiencies

D. limitations

33. A. cut

B. shortage

C. lack

D. drain

34. A. getting over

B. getting in

C. getting back

D. getting along

35. A. improve

B. enhance

C. develop

D. proceed

36. A. concerns b. issues

C. advantages

D. problems

37. A. still

B. always

C. habitually

D. consequently

38. A. call in

B. call over

C. call upon

D. call out

39. A. always

B. rarely

C. little

D. sometimes

40. A. little

B. enough

C. many

D. much

41. A. and

B. but

C. still

D. or

42. A. tolerant

B. hostile

C. indifferent

D. good-tempered

43. A. agreement

B. consensus

C. compromise

D. deal

44. A. go about

B. go over

C. go in for

D. go through

45. A. seldom

B. less

C. probably

D. certainly

46. A. dependent

B. a good source of information

C. of great value D.

reliable

47. A. familiar

B. cold

C. humid

D. new

48. A. though

B. while

C. since

D. as

49. A. enthusiasm

B. hesitation

C. caution

D. concern

50. A. same

B. equivalent

C. equal

D. similarity

PART IV GRAMMAR & VOCABULARY (15 MIN)

There are thirty sentences in this section. Beneath each sentence there are four words or phrases marked A, B, C and D. Choose one word or phrase that best completes the sentence.

Mark your answers on your answer sheet.

51. If you explained the situation to your solicitor, he ________ able to advise you much better than I can.

A. would be

B. will have been

C. was

D. were

52. ________, Mr. Wells is scarcely in sympathy with the working class.

A. Although he is a socialist.

B. Even if he is a socialist.

C. Being a socialist.

D. Since he is a socialist.

53. His remarks were ________ annoy everybody at the meeting.

A. so as to

B. such as to

C. such to

D. as much as to

54. James has just arrived, but I didn't know he ________ until yesterday.

A. will come

B. was coming

C. had been coming

D. came

55. ________ conscious of my moral obligations as a citizen.

A. I was and always will be.

B. I have to be and always will be.

C. I had been and always will be.

D. I have been and always will be.

56. Because fuel supplies are finite and many people are wasteful, we will have to install ________ solar heating device in our home.

A. some type of

B. some types of a

C. some type of a

D. some types of

57. I went there in 1984, and that was the only occasion when I ________ the journey in exactly two days.

A. must take

B. must have made

C. was able to make

D. could make

58. I know he failed his last test, but really he's ________ stupid.

A. something but

B. anything but

C. nothing but

D. not but

59. Do you know Tim's brother? He is ________ than Tim.

A. much more sportsman

B. more of a sportsman

C. more of sportsman

D. more a sportsman

60. That was not the first time he ________ us. I think it's high time we ________ strong actions against him.

A. betrayed…take

B. had betrayed…took

C. has betrayed…took

D. has betrayed…take

61. What's the chance of ________ a general election this year?

A. there being

B. there to be

C. there be

D. there going to be

62. The meeting was put off because we ________ a meeting without John.

A. objected having

B. were objected to having

C. objected to have

D. objected to having

63. ________ you ________ further problems with your printer, contact your dealer for advice.

A. If, had.

B. Have, had.

C. Should, have.

D. In case, had.

64. He asked me to lend him some money, which I agreed to do, ________ that he paid me back the following week.

A. on occasion

B. on purpose

C. on condition

D. only if

65. Children who stay away from school do ________ for different reasons.

A. them

B. /

C. it

D. theirs

66. –Why are you staring?

–I've never seen ________ tree before.

67. There are still many problem ahead of us, but by his time next year we can see light at the end of the ________.

A. battle

B. day

C. road

D. tunnel

68. We realized that he was under great ________, so we took no notice of his bad temper.

A. excitement

B. stress

C. crisis

D. nervousness

69. The director tried to get the actors to ________ to the next scene by hand signals.

A. move on

B. move off

C. move out

D. move along

70. His ideas are invariably condemned as ________ by his colleagues.

A. imaginative

B. ingenious

C. impractical

D. theoretical

71. Thousands of people turned out into the streets to ________ against the local authorities' decision to build a highway across the field.

A. contradict

B. reform

C. counter

D. protest

72. The majority of nurses are women, but in the higher ranks of the medical profession women are in a ________.

A. minority

B. scarcity

C. rarity

D. minimum

73. Professor Johnson's retirement ________ from next January.

A. carries into effect

B. takes effect

C. has effect

D. puts into effect

74. The president explained that the purpose of taxation was to ________ government spending.

A. finance

B. expand

C. enlarge

D. budget

75. The heat in summer is no less ________ here in this mountain region.

A. concentrated

B. extensive

C. intense

D. intensive

76. Taking photographs is strictly ________ here, as it may damage the precious cave paintings.

A. forbidden

B. rejected

C. excluded

D. denied

77. Mr. Brown's condition looks very serious and it is doubtful if he will ________.

________.

A. pull back

B. pull up

C. pull through

D. pull out

78. Since the early nineties, the trend in most businesses has been toward on-demand, always-available products and services that suit the customer's ________ rather than the company's.

A. benefit

B. availability

C. suitability

D. convenience

79. The priest made the ________ of the cross when he entered the church.

A. mark

B. signal

C. sign

D. gesture

80. This spacious room is ________ furnished with just a few articles in it.

A. lightly

B. sparsely

C. hardly

D. rarely

  • 29.8

    ¥45 每天只需1.0元
    1个月 推荐
  • 9.9

    ¥15
    1天
  • 59.8

    ¥90
    3个月

选择支付方式

  • 微信付款
郑重提醒:支付后,系统自动为您完成注册

请使用微信扫码支付(元)

订单号:
支付后,系统自动为您完成注册
遇到问题请联系 在线客服

常用手机号:
用于找回密码
图片验证码:
看不清?点击更换
短信验证码:
新密码:
 
绑定后可用手机号登录
请不要关闭本页面,支付完成后请点击【支付完成】按钮
遇到问题请联系 在线客服